Friday, March 08, 2013

God and the Big Bang

The Big Bang theory posits a temporal beginning to the universe, and doesn't explain why the Big Bang happened, or why the universe exists at all. Why is there a universe, as opposed to none? Couldn't it just as easily have been the case that nothing existed? Previous theories, like the Steady State or the Oscillating Universe theory, claimed that the universe has always existed. But these theories have been rejected. With the Big Bang, this is not the case. There was a beginning. So, even if you reject the Bible from the fourth word (In the beginning God), you have to say that the first three words are OK and in accordance with science.


If something begins to exist, and nothing caused it to exist, isn't that a strange thing to say? If we are eating lunch, and a bunny rabbit begins to exist and munches on your salad, would it make any sense to say "Oh, that rabbit just popped into existence out of nothing. It didn't have a cause."

259 comments:

1 – 200 of 259   Newer›   Newest»
Anonymous said...
This comment has been removed by the author.
BeingItself said...

"Couldn't it just as easily have been the case that nothing existed?"

I have no idea. Do you?

"If something begins to exist, and nothing caused it to exist, isn't that a strange thing to say?"

Who says the event did not have a cause? From what I understand, we don't know if the event had a cause. Besides, just because something seems strange to you, does not make it false.

Your rabbit example is a category mistake.

Son of Ya'Kov said...

The Oscillating universe is superficially like the Big Bang in the sense that in both scenarios the Universe starts out as an infinity dense point that surges and expands.

The difference is however the OU model postulates the Universe will somehow slow it's expansion stop then reverse and collapse in on itself.

It is my understanding based on the findings of the Hubble telescope the Universe is not likely to slow down for the Big Crunch to happen. Gravity is just not that powerful.

OTOH there are other Cyclical models that obtain the "bounce back" by other means.

From the standpoint of the First Way a possible Cyclical Universe is a non-starter.

Also interestingly Rabbinic Tradition states "God created and destroyed worlds before this one".

Martin said...

I think the Kalam argument is OK, but I think it is too dependent on contingent scientific facts. The Big Bang theory could be overturned, and what then?

I think Thomistic arguments are much better because they are more fundamental. Science will never prove that things don't change, and so will never overturn the first premise.

I think Feser is correct to say that natural theology should be grounded in metaphysics (philosophy of nature, to be precise), and not natural science.

Theists should check Kalam and ID.

Anonymous said...

After a period when the idea of an oscillating universe fell out of favor, it has been revived under the name of the cyclic model. The fact is, there is no real basis for saying that the universe actually had a beginning - let alone what might have caused it.

BeingItself said...

"I think Thomistic arguments are much better because they are more fundamental."

They may be "better" in the sense that no facts about the actual world could ever dissuade a committed Thomist.

If you just gin up a bunch of unfalsifiable principles, as the Thomist does, then anything follows.

But to the rest of us, grounded in reality, the arguments are miserable failures.

Son of Ya'Kov said...

BI

Ya gonna bore the crap out of us with your Positivism sans philosophy nonsense?

We already have Paps for that.

Papalinton said...

"With the Big Bang, this is not the case. There was a beginning."

This is somewhat of a misconception or misconstrual of the Big Bang theory. We only have information about what happened immediately after the big bang occurred; we only know and can only speak about the rapid inflationary period in which space, time, energy and particles begin to exist as separate and identifiably distinct states.

What we don't know as yet is the condition and state of the singularity, that deeply compressed point of energy, time, space, matter that constituted the singularity that made the big bang possible. No one has yet been able to gain any insight into what it was, how it got there, how long it may have existed prior to the big bang event. We don't know whether the singularity, out of which this universe began, was a product of an earlier process analogous to black hole formation. And if singularities are indeed at the centre of black holes, out of which space, time, matter and energy cannot escape, that suggests a multiplicity of singularities, because there are multiple black holes that have been identified throughout the observable universe. Singularities may exist eternally for all we know, subject to quantum fluctuations of as yet unidentified laws of physics coming into play, laws that triggered this particular singularity to reach the tipping point into the rapid expansion state resulting in realisation of the current universe.

"If something begins to exist, and nothing caused it to exist, isn't that a strange thing to say?"

This is a fairly low-level appeal to people's less than reliable intuitive sense, emotion and ignorance. Krauss clearly puts this to bed. He is rightly telling people that the universe came from nothing, but it is a quantum nothing. As we are now realizing, that quantum nothing is indeed full of something, idiomatically described as quantum energy, dark energy, dark matter, vacuum energy, etc that makes up around 96% of the universe. It is not a nothing akin to that found in a theist's brain. One can best analogise the difference between the science understanding of nothing and theological nothing as:

0 + 0 = 0; that is; nothing + nothing = nothing [theological]

1 + (-1) = 0; that is; one + (-one) = nothing [scientific) This conforms to every variant expression of the law of conservation of energy.

We know from naturalism that things exist, both seen and unseen. What we have no experience or knowledge about is, is the theological proposition of how nothing plus nothing equals nothing until god comes along as the unmoved-mover, the uncaused-cause. and in so doing we posit the equation; 0 + 0 + goddidit = universe. We have yet to deduce the function of this 'goddidit' mathematical symbol. And to date nothing, nothing, not even the small of an oily rag has provided any hint substantiating its existence. Indeed the gogdidit ideation has about the probability of being factual as a pink unicorn has of being a real beast.

The likelihood of Jesus-god being the progenitor of this universe simply fades evermore into the realms of fantasy and mythology.

Martin said...

BeingItself,

>If you just gin up a bunch of unfalsifiable principles, as the Thomist does, then anything follows.

Of course they are falsifiable. Re: the First Way, you need to show either that nothing is changing, or that if something changes it can change itself.

>But to the rest of us, grounded in reality, the arguments are miserable failures.

Oh, so I presume you can indeed show that one of those two premises is false.

BeingItself said...

Hi Martin,

You are shifting the burden of proof.

You might as well say "a god exists!", and then demand that I somehow show how such a thing cannot exist.

Son of Ya'Kov said...

In short Martin he can't because he would actually have to some reading and learning.

Papalinton said...

Victor
"So, even if you reject the Bible from the fourth word (In the beginning God), you have to say that the first three words are OK and in accordance with science."

So are the words, "Once upon a time ...."

Martin said...

BeingItself,

>You are shifting the burden of proof.

You said: "the arguments are miserable failures."

I asked you to support that claim with evidence. I note that you have not done so.

Doctor Logic said...

Thomism is a dead philosophy that imagines that almost every philosophical contribution since the Dark Ages never happened.

The First Way is a joke. It's an inductive conclusion, and a shabby one at that.

If you were born and lived on a raft in the middle of an ocean for your whole life, you might conclude that "shore" was an impossibility. Pervasively, the ocean surrounds you and always has. But this isn't a proof that there is no shore.

In the same way, causality is a feature of spacetime. And it's quite easy to conceive of an edge or beginning to spacetime, e.g., a first event at the Big Bang.

If this is the case, then the history of the universe is a giant 4-dimensional object which just is. It needs no creation because creation is something that happens in spacetimes, and there's no external space that needs to exist to contain our spacetime.

So, any mythical argument for the necessity of an external "first mover" is simply bunk. The history of the universe taken as a whole is a timeless entity. Time merely describes its internal structure.

Maybe there was something before the Big Bang. Maybe there are external spacetimes. It doesn't matter. The first mover argument will fail every time for the same reason: it's not necessary that the laws that are obeyed inside a space must follow on the boundary of that space.

It's true that if all causation came to an end, the world would be unintelligible. But if just some things can come into existence without a cause under certain conditions, then the world can remain intelligible and mostly explicable, and there's no inconsistency or contradiction.

Martin said...

DoctorLogic,

>The First Way is a joke.

Can you expand on this? What specifically is wrong with it?

Son of Ya'Kov said...

@Martin

You will have to forgive DL he obviously thinks the First Way is just another name for the Kalam Cosmological argument or something.

As you can tell Martin DL is ignorant of the brute fact Aquinas presupposed for argument's sake a past eternal Universe. Aquinas also believed it was not possible to scientifically or philosophically prove the Universe had a beginning.

We both know his Cosmological Argument does not involve a formal beginning of the Universe in spacetime but DL's knowledge of Atheist polemics obviously doesn't go above Dawkins or anti-ID fiat Creationist polemics.

I don't know...would it kill these Gnus and Gnu wannabe's to actually learn some philosophy?

Positivism, & Reductionist Materialism, are just the Atheist version of Young earth Creationism.

There are more intellectually respectible ways to be an Atheist.

@Dl

One name for you buddy.

Thomas Nagel! Burn your books by Dawkins and do your brain a favor.

Son of Ya'Kov said...

BTW to any Thomists reading this.

I'm taking bets DL is going to describe Aquinas' concept of motus(i.e. Motion) in terms of physical movement only and NOT in terms of any potency being made actual.

He will then invoke Newton as a possible defeater while being ignorant of the brute fact the concept of Motus in Aristotle is a metaphysical modeling and description of change not physical movement per say.

Oh and an additional 5 bucks he equates metaphysics with "spiritual" or the "supernatural" or some such ignorant blather.

So predictable......

grodrigues said...

@Doctor Logic:

So let us count the factual errors, misunderstandings, etc. on your post.

[About the First Way:]

1. "It's an inductive conclusion, and a shabby one at that."

Wrong; the First Way is a deductive argument.

2. "If you were born and lived on a raft in the middle of an ocean for your whole life, you might conclude that "shore" was an impossibility. Pervasively, the ocean surrounds you and always has. But this isn't a proof that there is no shore."

Invent a fallacious, ridiculous argument and then project it on your interlocutors. Right.

3. "In the same way, causality is a feature of spacetime. And it's quite easy to conceive of an edge or beginning to spacetime, e.g., a first event at the Big Bang."

First, emphatically, causality is not "a feature of spacetime". Second, you are confusing change (which does "need" time) with creation, which is *NOT* change. Third, whether space-time had a beginning (a space-time boundary) is *irrelevant* to the First Way.

4. "If this is the case, then the history of the universe is a giant 4-dimensional object which just is. It needs no creation because creation is something that happens in spacetimes, and there's no external space that needs to exist to contain our spacetime."

Wrong. Even if the universe is viewed as a 4d space-time manifold eternally existing, there is still need to explain why it exists, rather than just nothing at all. Whether our universe is part of a larger ensemble of universes, embedded in a larger space-time, or whatever, is irrelevant to the question; or more precisely, the question is just deferred to another level.

5. "The history of the universe taken as a whole is a timeless entity. Time merely describes its internal structure."

It hardly goes without saying that time is "internal" to space-time; there is no time "out" of space-time. But as I said before, this is irrelevant to the First Way. Aquinas famously argued that one cannot prove by reason alone that the universe had a beginning, or was past-finite. He did believe in that, of course, he just thought it was a piece of divine revelation, not something one could reason to by reason alone. I think he is wrong on this count; but the Master of Philosophers, Aristotle, did believe that the universe was past-eternal, so Aquinas concocted his arguments for the existence of God in a way that the question of whether the universe had a beginning or not is simply irrelevant.

6. "Maybe there was something before the Big Bang. Maybe there are external spacetimes. It doesn't matter."

What exactly is "before" the Big Bang? Are you thinking of cyclic scenarios?

7. "The first mover argument will fail every time for the same reason: it's not necessary that the laws that are obeyed inside a space must follow on the boundary of that space."

You are correct on the second sentence; incorrect on the first, because (and I am going to repeat myself) it is irrelevant to the First Way whether the universe had a space-time boundary; whether the physical laws break down past the boundary, etc.

8. "It's true that if all causation came to an end, the world would be unintelligible. But if just some things can come into existence without a cause under certain conditions, then the world can remain intelligible and mostly explicable, and there's no inconsistency or contradiction."

So let me get this straight. You acknowledge that doing away with causality is doing away with intelligibility (which I agree) but then go on claiming that "some things can come into existence without a cause"? I suppose that among said "things" is the entirety of the universe. But then according to you, it would then be unintelligible why the universe exists at all. So how exactly do you pretend to square this circle?

BeingItself said...

Martin,

How about this. You make an argument. If I find it persuasive, I'll let you know. If not, I'll explain why not.

Martin said...

BeingItself,

You said, "But to the rest of us, grounded in reality, the arguments are miserable failures."

I would like you to tell me why you think an argument such as the First Way is a miserable failure.

Unknown said...

It's a miserable failure because he wants atheism to be true, dislikes Christianity, or both.

BeingItself said...

Martin,

There are multiple versions. I don't know which one you are referring to.

Martin said...

BeingItself,

>There are multiple versions. I don't know which one you are referring to

There is only one version of the First Way. That was the argument I brought up and that you said is a miserable failure. I've asked three times now for you to support that statement.

Son of Ya'Kov said...

@DL & BI

Really would it kill you guys to actually learn some philosophy & get past this dogmatic kneejerk Positivism and anti-Fundamentalist Christian Polemics?

Or do you lot think looking ignorant on a subject & spouting uniformed mindless platitudes will somehow convince grod, Martin and or myself to have a "come to Science/Dawkins/Myers moment" where we will realize threw a burning in our bosom that God does not exist and Dawkins in our prophet?

Seriously is that your plan?

Anonymous said...
This comment has been removed by the author.
BeingItself said...

Martin,

Are you going to run the argument or not?

Son of Ya'Kov said...

>Ben this 'you are just fighting fundamentalists' rhetoric is itself a piece of empty rhetoric that contributes zero.

You full of it Zack. You are just being your own contrary "a pox on both your houses" self.

It's getting old.

BI is being a troll(because he has a history of it) and DL is making ignorant statements about Thomism.

Live with it.

Marcus said...

Why is the First Way an inductive conclusion? Just examine the specifics of the argument and it should remind you how much deduction depends on induction.

The first two premises, that some things are in motion (or change, it doesn't really matter) and that a thing can't move itself, are observational truths. They are entirely in the realm of induction. So you could say they are not inductive premises but then we would have nothing more to go on than Aquinas' word that this is how reality works outside of our inductive experiences, as literally all the evidence we have to support these premises is inductive. What do we know about a-temporal causality (if that even makes sense to say) which this argument is attempting to argue for? Nothing.

The same basic problem goes for Kalam. All the evidence we have to support the contention "whatever begins to exist has a cause" is inductive. The evidence here is overwhelming but it's all for physical creation ex materia happening in spacetime while the argument is making the case for nonphysical creation ex nihilo outside of spacetime. So the argument, at best, is just guessing in the dark about how such a process could work.

Son of Ya'Kov said...

>Are you going to run the argument or not?

@Martin

Don't waste your time. He is an old troll. He won't answer your questions or interact with you in good faith.

DL at least will try to debate you.
I'll grant him that.

Son of Ya'Kov said...

@Doubting Marcus

Aquinas is a realist and his arguments presuppose a moderate realist philosophical underpinning.

Your are presupposing a non-realist grounding. Which is fine but you have to make a positive case for either conceptionalism or nominalism and fend off all the criticisms that can be heaped on them.

One obvious problem I see here with you trying to Ad Hoc reduce all arguments to the level of the inductive to maintain your skepticism is the very argument you are proposing would itself be inductive and thus we must be skeptical of it.

At least your trying to argue from philosophy. I'll give you that.

Cheers.

Son of Ya'Kov said...


>So you could say they are not inductive premises but then we would have nothing more to go on than Aquinas' word that this is how reality works outside of our inductive experiences, as literally all the evidence we have to support these premises is inductive.

No Aquinas presuposes what appears in our intellect is first present in our senses.

If you wish to doubt our senses that is fine but then how will you be able to know anything?

Plus how will you be able to know your inital argument is valid?

Papalinton said...

DL is right on the money. Aquinas only remains on the philosophical agenda simply by virtue of being inextricably bound to the christian mythos. A-T philosophy makes ripples only in the theological pond. It is of no consequence or relevance to the broader philosophical challenges going forward.

Aquinas's Five Ways is intuitive rationalization in the absence of scientific knowledge. Aquinas has been refuted so many times it has become rather tiresome. Contemporary philosophy has largely set aside A-T philosophy, as a time worn intellectual product which must be read within an historical context and within the social, cultural and political milieu of its day. It can only be read as a universal 'truth' within the limiting parameters of christian theology which itself is now conclusively found to be explanatorily without foundation other than as a cultural phenomenon and artefact.

A simple line of rational queries on the a priori reasoning of Aquinas easily calls into question some of Aquinas's presumptions:

"Another important point to make is that Aquinas says that nothing can be infinite, and then goes on to say that God is infinite. He says that nothing is the cause of itself, then says that God is the cause of Himself. Admittedly, he does say that God is outside the universe. Perhaps this means he is not bound by the limitations which apply within it. Even so, this involves conceding that something can be infinite or indendent, undermining Aquinas' basis for arguing that the universe must have a cause. And we can question whether it makes sense to speak of something outside the universe. If not, the response I have offered to Aquinas is an appeal to mystery, or - less charitably - a dodge." See HERE.

Need I go further?

Thomists; you must fast forward yourselves around a 1,000 years if you are going to make any relevant contribution to society.

Unknown said...

Here's the First Way:

1. Evident to the senses is motion. (Premise)

2. Everything in motion has its motion sustained by another. (Premise)

3. Either an Unmoved Mover exists, or else there is an infinite regress of sustaining movers. (Implied by 1 and 2)

4. There cannot be an infinite regress of sustaining movers. (Premise)

5. Therefore, an Unmoved Mover exists. (From 3 and 4)

Thomas defends each of the argument's premises, and most (if not all) objections to the argument are based on misinterpretations of both Newtonian physics and Aristotelian metaphysics.

If someone can state clearly which premise(s) is wrong, then I'd be happy to constructively debate the matter.

Marcus said...

BenYachov,

Forget labels, I'm not assuming large baggage nor am I pulling out radical skepticism. I do think the objects of deduction are known from induction but that isn't a problem for using deduction. The problem with this argument is a fallacy of composition. You can't go from facts about objects based entirely on experience to conclusions allegedly independent of that experience which produced the entire set of facts. What is true of the part, is not necessarily true of the set.

To be specific, after saying some things are in motion and a thing can not move itself the claim is made that an infinite regress is impossible. What's wrong with this claim is he's using knowledge about parts he observes to make a statement about the set to which those parts belong.

The fact that rocks or trees didn't always exist or can't move themselves doesn't in the least imply the entire universe has those properties. Aquinas and all other cosmological proponents are saying all objects inside the universe behave like this, therefore the universe itself is subject to those rules.

The simple fact one can't imagine a universe which has always existed or an undending chain of causation, doesn't mean we can dismiss either as a possibility. Whatever the answer is to the origin of existence it will be mind-boggling so you can't use mind-boggling-ness as a reason to dismiss a answer.

Doug Benscouter,

4 is unjustifiable. See above.

grodrigues said...

@Doubting Marcus:

"The fact that rocks or trees didn't always exist or can't move themselves doesn't in the least imply the entire universe has those properties."

This is a common misunderstanding of the First Way. There is absolutely no need to invoke the universe or the "whole", whatever you take that to be. Any garden variety object of our experience (a tree, a rock, etc.) is sufficient, because what Aquinas is getting at is that God is the First Mover, first in the ontological sense not the temporal one, for *any* causality at all to happen in the universe in the *here and now*, not in the sense of knocking down the first domino in a long chain of dominos and get the ball rolling.

note: this is not relevant to the First Way, but it is relevant to cosmological arguments that do argue from the totality of the universe; the composition fallacy is an informal one. There are valid inferences that proceed from the parts to the whole. For example, if every brick in the wall is red, then the wall itself is red.

Unknown said...

Hi Doubting Marcus,

I don't think it's accurate to say Thomas is committing a composition fallacy. In fact, his arguments against an infinite regress have nothing to do with relating the parts to the whole.

One of Thomas's arguments is that it is impossible to form an actual infinite by successive addition whenever one begins counting. So, even if the universe's past is infinite, it is still composed of finite parts. Now, at each finite period of time, the regress of sustaining movers begins anew: 0, 1, 2, 3 . . .

It is always and indefinitely possible to count another number before arriving at infinity, which means the regress of sustaining movers (moving in any finite period of time) must be finite.

Martin said...

BeingItself,

So you know that the argument is a miserable failure, but you have no idea how it goes.

Interesting.

grodrigues said...

@Doubting Marcus:

Ack, typo. Sorry. Replace "for *any* causality at all to happen in the universe in the *here and now*" with "for *any* change at all to happen in the universe in the *here and now*".

Marcus said...

grodrigues,

I don't see how specifying the argument isn't temporal helps. The justification for the premises are temporal so if you remove that you're left with no justification to believe any of it. The argument is still going from the observed behavior of specific garden variety objects to the nature of all possible objects. That's an unbridgeable gap without some evidence as to why we should believe this.

As an aside, yes the fallacy of composition does not demonstrate the conclusion is necessarily wrong. However, the way we decide when it is truly a mistake is to look and see. Since there really isn't a way to do that for the totality of existence then, as I said before, such arguments are just guessing.

Marcus said...

Doug Benscoter,

The problem of composition I'm talking about is the gap between saying our "in my observations of the world, x is true" to "x is true about the universe." However, put that aside.

I agree with you that you can't get to infinity by counting. Now tell me how this proves the universe is finite. All it seems to prove, in this argument, is that infinity behaves in non-intuitive manners.

1) You can't count to infinity.
2) ???
3) The universe is temporally finite.

That's not an argument. Appealing to our intuition here is useless as the alternative offered, that something always existed, is just as counter-intuitive. Reality has no obligation to fit nicely with our intuition.

Not to mention you can't simultaneously argue that the universe can't have an infinite past because such a feat is impossible while simultaneously arguing that a god does have an infinite past. That's just self-refuting.

Unknown said...

Marcus, I think you misunderstand what the argument shows. It is not intended to show that the universe's past is finite. Rather, it is intended to show that the regress of sustaining causes, not originating causes, is finite. Even if the universe's past is infinite, it is still composed of finite intervals. At each finite interval, the regress of sustaining movers begins anew, as I mentioned. It's this type of regress that demonstrably cannot be infinite.

BeingItself said...

"So you know that the argument is a miserable failure, but you have no idea how it goes."

I have encountered numerous arguments people label "The First Way". All of those have been pathetic. Perhaps yours won't be.

So, let's have it.

Marcus said...

Doug Bencoster,

I think I understand quite well what the argument is trying to say, I just find it wanting, but perhaps we are having semantic difficulty.

To summarize the argument in a different way: We only observe contingent events. Contingent events can't cause themselves. A string of contingent events can't go on forever into the past because, as he originally said, "whatever is in motion must be put in motion by another. If that by which it is put in motion be itself put in motion, then this also must needs be put in motion by another, and that by another again. But this cannot go on to infinity, because then there would be no first mover, and, consequently, no other mover."

This is to say, at some point in the past something started the chain of contingent events -- and it can't be one of the contingent events! -- otherwise there would be no such chain.

I'm saying:

1) It is an unjustified extrapolation from what we observe about contingent events to the total set of contingent events (or, if you prefer, "sustaining causes"). As Doctor Logic noted above, that's like saying a person born on a raft claiming because they've never seen the shore it is impossible. At best, this is an appeal to our intuition.
2) You can't argue it is impossible for the universe itself to be origin of the chain but ok for a god to be so. To do so would be to beg the question.

Martin said...

Doubting Marcus,

> A string of contingent events can't go on forever into the past

No. That is exactly what it doesn't say. Aquinas specifically eschewed the possibility of proving that the universe had a beginning, and the First Way is built on Aristotle's unmoved mover, which not only didn't argue for a beginning to the universe but in fact begins by proving that the universe is eternally old.

The First Way is arguing for a presently active cause of all change.

I find the best way to explicate it is in terms of a lamp in a living room. The lamp can't power itself, but it is lit. So the power must be coming from somewhere else: the electrical outlet. But the same reasoning applies to the outlet: it can't power itself either, so it must be getting it's power from somewhere else: the power lines.

And so on.

Note that each of these members are all receivers of electricity, so somewhere on the other end of the chain must be a giver of electricity. It makes no sense to have receivers but no giver. In this case, the power plant. The power plant gives electricity without having to be powered by anything else.

But of course that's not strictly true. The power from the power plant comes from the burning coal, which depends on the mass and atomic structure of the coal, and so on. But now you leap out of the analogy and into the argument proper. There must be something actuating this whole chain that does not need to be actuated by anything further.

Unknown said...

Marcus, I'm afraid you still misunderstand the argument. The finitude or infinitude of the universe's past has nothing to do with the First Way. Let's start off with a simple observation. Do you recognize that there's a difference between an originating cause and a sustaining cause?

Marcus said...

Martin,

There's little difference in kind between your analogy and my description of sustained causes as contingent facts. You could call the pieces in that analogy, the lamp and the electrical outlet, contingent facts. You are saying looking at those facts tells us this chain must have begun somewhere, a chain of such contingent facts can't go on indefinitely, they need a non-contingent fact to have started them. In the analogy, the power plant, in Aquinas' argument, god, the originating cause, and the sustaining cause as well keeping them in order.

At the risk of repeating myself, I'm saying you can't make that leap from "everything I observe within Set A needs x" to "Set A needs x." The need for sources of movement for such things as lamps is something we observe within a set, the universe, but we can't just assume because it applies inside the set it applies to the set itself. That's what this argument is doing and that's what I'm objecting to. Note that this holds whether or not the universe is infinite.

Martin said...

Doubting Marcus,

>they need a non-contingent fact to have started them

There is a considerable difference between saying a non-contingent fact started them, implying something in the past, and saying a non-contingent fact sustains them, implying something that is still operative. You mentioned the past, and my point was that the argument has nothing to say about the past.

>I'm saying you can't make that leap from "everything I observe within Set A needs x" to "Set A needs x."

The argument never makes that move, so I don't know why you are attacking it. The argument says that it is logically impossible for something to be both actually X and potentially X at the same time. This is the law of non-contradiction. And that therefore something cannot be both changer and changed. At no point does it suggest that we observe things needing a cause, and that therefore this must apply to the universe as a whole. Rather, take any example of change: the ice cubes in your freezer. That's all the argument requires as the first premise. The second premise is the law of non-contradiction: the ice cannot be both actually ice, and potentially ice (i.e. liquid) at the same time. So it cannot be the cause of its own change. So something else must change it. And from that it follows that there is an unchanging changer that is ultimately responsible for the ice cubes in your freezer.

Marcus said...

Martin,

"The argument never makes that move, so I don't know why you are attacking it... At no point does it suggest that we observe things needing a cause, and that therefore this must apply to the universe as a whole"

I thought there was some confusion before but now I know I question if we even speak the same language.

There would be no need for a claim of a god were it not for the contention the universe can't sustain itself. If you are conceding that yes indeed the universe can sustain itself, and that it doesn't need an originating mover, then you have already conceded the point as there is no need to appeal to a god. If you are saying the rules which apply to the objects we observe in the universe, don't necessarily apply to the universe itself then there is no disagreement and again there is no need to invoke an unmoved mover.

Papalinton said...

Doug Benscooter
Re-iterating Aquinas's First Way ad nauseam since the Middle Ages simply does not make it any more substantive.

Professor Richard E Hennessy from the Catholic, Merrimack College, has refuted Aquinas's First Way noting it as little more than intuitive rationalizing. Indeed he does it through contesting Feser's somewhat blinkered view of Aquinas in TLS. The refutation is robust and indelibly casts doubt on the a priori suppositions that Aquinas posits to make his explanation work. But I'll let Hennessy talk for himself: See his refutation HERE.

Anonymous said...
This comment has been removed by the author.
Anonymous said...

Doug,

Didn't Thomas Aquinas say that only an actual motion can convert a potential motion into an actual motion?

It seems to me that your statement #3 "Either an Unmoved Mover exists, or else there is an infinite regress of sustaining movers. (Implied by 1 and 2)" is a non sequitur. Something that is unmoved cannot cause motion.

Martin said...

Doubting Marcus,

>There would be no need for a claim of a god were it not for the contention the universe can't sustain itself.

The argument does not make any claims about the universe. All it requires is one example of something changing. The water in your freezer becoming ice cubes. From this it follows that an unchanging changer must be the ultimate cause of that event.

Martin said...

Zach,

>This is open to debate

It isn't just Feser. Copleston says the Thomistic arguments are hierarchical or ontologically ordered, and not temporally. The IEP says it. Gilson says it. Martin says it.

And Aquinas says it: "In efficient causes it is impossible to proceed to infinity "per se"--thus, there cannot be an infinite number of causes that are "per se" required for a certain effect; for instance, that a stone be moved by a stick, the stick by the hand, and so on to infinity. But it is not impossible to proceed to infinity "accidentally" as regards efficient causes..."

I don't see much debate, at least among the main scholars including the man himself.

Anonymous said...
This comment has been removed by the author.
Anonymous said...
This comment has been removed by the author.
grodrigues said...

@Doubting Marcus:

"I don't see how specifying the argument isn't temporal helps."

Simply because the nature of the argument does *not* involve temporal relations essentially, but rather the distinction between act and potency and the principle of causality (in the Scholastic formulation).

"The argument is still going from the observed behavior of specific garden variety objects to the nature of all possible objects."

What do you mean by "the nature of all possible objects"? It is true that some claims are made about all beings (e.g. the reality of the distinction between potency and act) but they are not arrived at by the argument itself but *reasoned to* by independent considerations from the philosophy of nature.

" That's an unbridgeable gap without some evidence as to why we should believe this."

We have evidence; e.g. in the form of arguments.

"However, the way we decide when it is truly a mistake is to look and see. Since there really isn't a way to do that for the totality of existence then, as I said before, such arguments are just guessing."

First, "look and see" is emphatically *NOT* the only way to decide. The way to decide is to make an argument, which may, and usually does, involve some sort of empirical input. But this input can be as general as change is real. Second, and I will repeat myself, some cosmological arguments do argue from the totality of existence, but that is not the case of the First Way, so why do you keep harping on this is beyond me.

[to Doug Benscoter:]

"To summarize the argument in a different way: We only observe contingent events."

For the love of God, the First Way is *not* about contingency. The Third Way does deal with the distinction between contingent and necessary beings.

"It is an unjustified extrapolation from what we observe about contingent events to the total set of contingent events (or, if you prefer, "sustaining causes")."

The First Way does not invoke "the total set of contingent events", so this is just irrelevant. For the cosmological arguments that do involve such, the extrapolation is not unjustified, because arguments are given to make the extrapolation. The arguments may be wrong, but they are still there and claiming that they are not is just ignorance.

"You can't argue it is impossible for the universe itself to be origin of the chain but ok for a god to be so. To do so would be to beg the question."

No, no, no, thousand times no. This once again, betrays a misunderstanding of what Aquinas is getting at. There is absolutely no question-begging going on; the universe *CANNOT* be the origin of the chain (origin in the relevant ontological sense), because the universe itself is a composite of act and potency. It is precisely because God *is* pure act that He can be, and is the origin of the chain.

grodrigues said...

@im-skeptical:

"Something that is unmoved cannot cause motion."

Wrong. Unmoved in this context means being pure act, or totally lacking in what is known as passive potency or being acted on.

Marcus said...

Martin,

"The argument does not make any claims about the universe. All it requires is one example of something changing. The water in your freezer becoming ice cubes. From this it follows that an unchanging changer must be the ultimate cause of that event."

If you extend that argument back, instead of just thinking "and so on" it follows that what he is talking about is the set of all things in motion, which he says need some originating mover. But what is the set of all things in motion? The universe.

Take your electricity analogy. He's saying the lamp depends on the outlet, which depends on the power plant, which depends on fuel, which was ultimately created by energy from the sun, the sun's motion came from hydrogen atoms and the hydrogen atoms motion from the big bang. And that is where he says there must have been a first mover (though, obviously, he knew nothing of the big bang). The set he contends needs explaining is the universe itself. If the motion of the universe doesn't need explaining then, as I said, the argument is pointless as there's no need to say there was a first mover to start the motion in the universe which lead to, among other things, the lamp on the table.

BeingItself said...

When are you guys going to realize that all this yammering about something as "pure act" and and another as "composite of act and potency" is just a bunch a baloney.

Your opponents are arguing from things we know about the real actual existing world, while you guys are arguing from principles you pulled out of your ass.

There is just no common ground here to have a conversation.

Martin said...

Doubting Marcus

>which depends on fuel, which was ultimately created by energy from the sun, the sun's motion came from hydrogen atoms and the hydrogen atoms motion from the big bang.

Aquinas distinguishes between a series ordered essentially, and a series ordered accidentally. In the first chain, each member is dependent here and now on the next. Remove any member, and the effect stops. Abstractly:

A causes C through intermediate B, such that if A stops, then C stops.

A series ordered accidentally does not require each member to be in place anymore. Abstractly:

A causes B, then B causes C.

The chain you are talking about leaps into the accidental series, when you talk about the creation of the sun in the past, etc. The past doesn't need to be around anymore to make the present work. All Aquinas is interested in is what is required to be in place right now for a certain effect to work:

1. Water
2. Freezer
3. Electricity
4. Power plant
5. Coal
6. Atomic structure that allows combustion to occur
7. ???

Unknown said...

Papalinton, we've also cited propositional logic "ad nauseum" since Aristotle. Should we abandon that, too?

Unknown said...

im-skeptical, Thomas stated not that only an actually moving entity can actualize a potentiality, but that only an actuality can actualize a potentiality. There's a huge difference.

Something immutable can change another. Imagine a beautiful painting. The observer is said to be "moved" by its beauty, even though the painting need not be in motion. Rather, the painting moves a thing as an object of desire. The Unmoved Mover is the supreme object of desire, being God's character.

Marcus said...

grodrigues,

"For the love of God, the First Way is *not* about contingency. The Third Way does deal with the distinction between contingent and necessary beings."

The first, second and third of Aquinas' arguments are functionally the same. In this case to say an object's motion depends on some other object to have started it is to say it is, quite literally, to say it is contingent. Whether or not you are holding that something is contingent because all movement (or change) must have been sparked, because all causes must have a first cause, or if you explicitly declare these things to be contingent you are making the same argument. That's because to say something is contingent is to say it is dependent on something else. So saying some object's motion is dependent on some other object's motion, which ultimately must be dependent on some entity which is not, in that way, dependent is to argue for a non-contingent force. In the case of the first way, a non-contingent originator of motion. That's what a first mover would be, a force of motion not which didn't have that dependency.

"There is absolutely no question-begging going on; the universe *CANNOT* be the origin of the chain (origin in the relevant ontological sense), because the universe itself is a composite of act and potency. It is precisely because God *is* pure act that He can be, and is the origin of the chain."

How do you and Aquinas know the universe isn't pure? Why not apply the same rules about the motion observed in the universe to the god you are invoking? After all, if god sustains the motion we see surely, by this same reasoning, his motion must have come from somewhere. But of course, I suspect I know what the reply is to that might sound like "NO, NO, NO! God is the first mover, unlike the universe itself the rules of causation I just argued for don't apply to god." And likely by definition.

"The First Way does not invoke "the total set of contingent events", so this is just irrelevant. For the cosmological arguments that do involve such, the extrapolation is not unjustified, because arguments are given to make the extrapolation. The arguments may be wrong, but they are still there and claiming that they are not is just ignorance."

Where did I deny arguments exist attempting to bridge this gap? I'm denying they are effective because, for the millionth time, to observe the behavior of parts within the universe and then to claim the universe itself must have the same properties as it's parts is wrong every time. These arguments are all claiming "Because of what I observe within the universe, I can't imagine the universe itself having property X." Where X is not needing an external force which started its movement, its chain of causation, its contingent objects.

Marcus said...

Doug Benscoter,

"...
6. Atomic structure that allows combustion to occur
7. ???"

Why the atomic structure is the way it is and not some other way is a property of the universe itself. Again you are back to the property of the universe itself needing an explanation. It doesn't matter if you go back in time or down to the most fundamental nature of what we see, you are still getting at the same erroneous claim that "the universe itself must have an origin of motion external to the universe itself and I know this because from my experience in the universe I can't imagine it being otherwise."

Perhaps more importantly, as BeingItself noted, this seem fruitless. I've long since tired of repeating myself (which I'm sure someone will henceforth claim is because I'm not comprehending your claims or feigning ignorance). At this point, anyone reading this can judge for themselves.

Unknown said...

Marcus, I think your response was to someone else. I never mentioned the atomic structure of the universe. More importantly, I reiterate, the First Way has nothing to do with the origin of motion, but only sustaining motion.

Do you really deny, as BeingItself seems to, that things exemplify potentiality and actuality, e.g. that an acorn is merely an acorn in actuality but an oak tree in potentiality?

Marcus said...

Doug,

Ah yes, my mistake, that was for Martin. Sorry for the confusion.

"Do you really deny, as BeingItself seems to, that things exemplify potentiality and actuality, e.g. that an acorn is merely an acorn in actuality but an oak tree in potentiality?"

No, but I don't think he's denying an acorn can potentially be a tree but is just an acorn currently. However, my point about repeating myself still stands. I really wouldn't have replied at all had I not made that mistake with names. Perhaps another time.

Unknown said...

No worries, Marcus.

grodrigues said...

@Doubting Marcus:

"The first, second and third of Aquinas' arguments are functionally the same."

No, they are not. But this is not the most important point of discussion, so I am dropping it.

"How do you and Aquinas know the universe isn't pure?"

You are asking how does Aquinas knows that the universe is not pure act? Do *you* know what Aquinas means by being pure act? If you did know, you would also know why the universe is not, *cannot be*, pure act, or for that matter, even a necessary being.

"After all, if god sustains the motion we see surely, by this same reasoning, his motion must have come from somewhere."

The *conclusion* of the argument is that there must be an Unmoved Mover to explain the fact that any change at all is happening in the universe. But the Unmoved Mover cannot be, not even in principle, moved by another, because that is just the conclusion of the argument that there must be an Unmoved Mover to sustain it all, so your objection is just incoherent.

"I'm denying they are effective because, for the millionth time, to observe the behavior of parts within the universe and then to claim the universe itself must have the same properties as it's parts is wrong every time."

And for the millionth time, the First Way (or any of the Five Ways actually) does not proceed from the behavior of the proper parts of the universe to the universe as a whole, although some *other* cosmological arguments do. For the millionth time, no *one* is claiming that "Because of what I observe within the universe, I can't imagine the universe itself having property X", that is just your fallacious, ignorant rendering of what defenders of the arguments actually hold.

Anonymous said...

grodrigues,

"Wrong. Unmoved in this context means being pure act, or totally lacking in what is known as passive potency or being acted on."

I understand that the god of Thomas is defined as pure actuality, but isn't it a rather convenient definition. Movement is defined as transforming from potency to actuality, but something unmoved is also actuality. Those things don't agree with each other. It still seems like a contradiction to me.

Anonymous said...

Doug,

Thank you also for replying to my question. Do you think that the empty vacuum of deep space is actuality? I think it would have to be if Thomas' view of the world is correct, and yet it seems to me that there's nothing there that could cause anything to happen.

Unknown said...

im-skeptical, no problem. Call me a philosophy geek, but I live for these types of discussions. :)

As Christopher Ray points out, "it is a mistake to think of any physical vacuum as some absolutely empty void." (Time, Space and Philosophy, Routledge, 1991, p. 205.) Rather, vacuums are composed of a rich fluctuating energy. Moreover, even a hypothetically empty space has the potential to be filled, and so it stands in causal relations.

Anonymous said...

Doug,

I'm not talking about quantum vacuum. I'm talking about the behavior of an ordinary object when placed in a real vacuum, far from the influence of any other objects.

We have used the example of water being moved to become ice, and in our example, it is the freezer that is the mover. I maintain that that's wrong. Physics tells us that it isn't some external thing that causes water to freeze, but it is instead some external thing that makes it stay in the form of water - namely heat that radiates or is conducted to the water. Take that away, and the water freezes. This can be illustrated by the example of deep space. Put a jar of water there, and it will freeze as its own heat radiates away.

The point of this is that there is nothing external to the water that moves it as it turns to ice. It does that on it own - just obeying the laws of physics. In fact the very notion of movement from potency to actuality is not consistent with physical reality as we understand it today.

That's why is is so difficult for me to accept the whole A-T bit. Aristotle was brilliant, but his understanding of how things work was severely lacking by modern standards. Yet his metaphysics was based on the understanding he had.

Unknown said...

The laws of physics are sufficient to account for the type of change your talking about, then. If the laws of physics are removed or suspended, then water will neither freeze nor melt. I sympathize with your reservation, but I think it's much simpler than you're making it out to be.

William said...

Aristotle felt that all causes must come to an end, with the Prime Mover. Sounds as if others (non A-T theorists) here think all explanation must come to an end, with the laws of physics.

But must there be either a cause or an explanation for everything?

Unknown said...

William, it's interesting to me that many atheists and pantheists appeal to the laws of nature as immutable, making nature a kind of substitute for the Unmoved Mover. Immutability points to pure actuality, so there can be no potential to change. Whether or not everything has to have an explanation (PSR) isn't a necessary point of debate in the argument from motion. So long as motion has an explanation, it doesn't follow that the Unmoved Mover also has an explanation.

Of course, for those of us who accept the PSR, such as myself, we hold that the Unmoved Mover is explained by necessity.

Anonymous said...

"If the laws of physics are removed or suspended, then water will neither freeze nor melt."

I take that to mean that the unmoved mover is acting directly on the water - the act of creation produces the laws of physics and thus moves the water directly. But if that's the case, why do we bother with different causes, like formal and material, and even efficient causes? All we need is an unmoved mover and the laws of physics. The rest is fluff.

Unknown said...

That's not entirely the case. God (the Unmoved Mover) sustains the laws of nature, and these laws act as instrumental causes of motion. Even with the Unmoved Mover as the ultimate source of all motion/change, there are still instrumental movers other than the laws of nature and different types of causes. I think your conclusion is a non sequitur. Did I misunderstand you? It wouldn't be the first time I misunderstood something. :D

Anonymous said...

"there are still instrumental movers other than the laws of nature and different types of causes"

Well, the example I gave was intended to show that there is no instrumental mover. (You'd call that efficient cause, wouldn't you?) There is no chain of causality at all. The water freezes of its own accord, without being influenced or moved by anything at all.

Unknown said...

The laws of nature are instrumental causes. The physical space that the water takes up is an instrumental cause. You can't take any the movers and still have motion.

Unknown said...

*You can't take away any of the movers and still have motion.

Anonymous said...

"The laws of nature are instrumental causes."

So it really does just boil down to an unmoved mover who creates the laws of physics, and they cause everything to happen. I say that because everything we see behaves according to the laws of nature, and no further explanation is needed for why things "move".

Martin said...

im-skeptical,

>This can be illustrated by the example of deep space. Put a jar of water there, and it will freeze as its own heat radiates away.

Note that you said "put" a jar of water in deep space.

Exactly right. For the water to change from liquid to solid, it must be placed there by something already actual: the airlock of the spacecraft. And for the airlock to change from closed to open, the switch must be flipped. And for the switch to be flipped, the fingers must flip it. And for the fingers to flip it, the muscles must move. And for the muscles to move, motorneurons must fire.

You see how this is one big chain? And on the other end, there must be something that causes actualization without having to BE actualized by anything further, otherwise you just would have this activity.

Unknown said...

im-skeptical, as I mentioned, atheists and pantheists tend to treat the laws of nature as immutable, which is nothing more than an Unmoved Mover substitute. The laws of nature (or Nature) would have to be pure actuality. Either way, there is an Unmoved Mover. I just don't share the view that the laws of nature are immutable, even though they exhibit regularity for the most part.

Anonymous said...

Doug,

"I just don't share the view that the laws of nature are immutable, even though they exhibit regularity for the most part."

Even if they are not immutable, it would still be fair to say that everything acts in accordance with them. So, say the creator wants to make a miracle by momentarily changing the law, the fact remains that everything that happens is a consequence of those laws.

Martin may ask, what is the cause of a specific event? "the airlock of the spacecraft. And for the airlock to change from closed to open, the switch must be flipped ... "

But it isn't so straight-forward. In reality, the entire history of the universe affects everything that happens now. For an acorn to become a tree, it takes sun, soil, weather, and the city was going to build a parking lot there, but the planner got killed in a horse-riding accident, which happened because it rained that day ... ... and because this one atom was in just the right position a billion years ago, and every other atom was also in just the right position, everything turns out just so today.

The point is that there is no such thing as a chain of causality. But everything does behave according to natural law.

Martin said...

im-skeptical,

>For an acorn to become a tree, it takes sun, soil, weather, and the city was going to build a parking lot there, but the planner got killed in a horse-riding acciden

Past events. Not essential to be in place right now as the cause of the water's freezing. If you remove the airlock, or the hand, or the switch, the water won't freeze.

William said...

" and because this one atom was in just the right position a billion years ago, and every other atom was also in just the right position, everything turns out just so today. "

Wow, what faith. Believing this takes more faith in your deterministic laws of nature than I have.

Papalinton said...

Doug
"Papalinton, we've also cited propositional logic "ad nauseum" since Aristotle. Should we abandon that, too?"

Did you end up reading Hennessy's paper? If not why not? And if so, perhaps you could let me know your response to his arguments. We may have a basis for further conversation.

About propositional logic? Yes we should abandon them if they have been found less than sufficient to prosecute an argument, because clearly propositional logic is only as good as, and relies utterly dependent and captive to the veridicality of the hypothesized premise; and if that initial premise is false, propositional logic, itself being simply a process, will not flush out the falsity of the premise, but carry it through to an 'apparent truth' of its conclusion.

HERE is an excellent peer reviewed paper on the process of 'propositional logic'.
In part it notes , "The most thoroughly researched branch of propositional logic is classical truth-functional propositional logic, which studies logical operators and connectives that are used to produce complex statements whose truth-value depends entirely on the truth-values of the simpler statements making them up, and in which it is ASSUMED that every statement is either true or false and not both. [My bolding and capitals]

Cheers


Anonymous said...

William,

"Wow, what faith. Believing this takes more faith in your deterministic laws of nature than I have."

OK, even if you deny determinism, don't you think causality looks more like a matrix than a chain?

Papalinton said...

Ben
"William, it's interesting to me that many atheists and pantheists appeal to the laws of nature as immutable, making nature a kind of substitute for the Unmoved Mover. "

Allow me to butt in. Wrong. Not immutable but simply testable, repeatable, verifiable, consistent with current knowledge and understanding, open to falsifiability and improvement. These are precious values that are not found in the immutable notion of the Unmoved Mover. The Unmoved Mover is a conversation and research stopper, the end of the line, the cul-de-sac of everything, a dead end. Period.

It is the unwarranted and unsupportable proposition of an immutable God that is the problematic stone wall barring the way to knowing and understanding everything we see, experience and relate to about the world, about the universe, about the cosmos. To accept such a proposition is to concede that all that has been accomplished during the past 2,100 years has been for nought because the truth about us, the world, the universe and the cosmos had been discovered, settled and bedded down at the time of the iron-age.

This is the form of 'propositional logic' you are asking all to believe and subscribe to? I'm afraid that argument simply does not carry any weight as a do-able example of propositional logic by any stretch.

ozero91 said...

If the universe was pure act, there would be no change, because nothing in the universe would have the potential to be anything else.

ozero91 said...

I don't see why picturing causality as a web rather than a chain changes anything. You still have act and potency.

ozero91 said...

"So it really does just boil down to an unmoved mover who creates the laws of physics, and they cause everything to happen. I say that because everything we see behaves according to the laws of nature, and no further explanation is needed for why things "move"."

But aren't the laws of nature just mathematical descriptions of the behavior of matter? Your statement amounts to "the behavior of matter is due to our mathematical descriptions of the behavior of matter." Except there is a problem: a mathematical description cannot cause anything.

Anonymous said...

"I don't see why picturing causality as a web rather than a chain changes anything. You still have act and potency."

But what you don't have is Aristotle's pre-scientific physical laws governing our modern view of how things work. Act and potency are very much part of his metaphysical world-view that grew out of that same pre-scientific understanding.

I'm still trying to keep my mind open about this A-T philosophy. I'll read and learn what I can about it, I'll ask for answers, but it's going to take some powerful logic that I have yet to see.

ozero91 said...

"But what you don't have is Aristotle's pre-scientific physical laws governing our modern view of how things work. Act and potency are very much part of his metaphysical world-view that grew out of that same pre-scientific understanding."

IIRC, the act/potency distinction is an explanation for why any sort of change is even possible. See Aristotle vs. Parmenides. Yes, Aristotle's "science" was pretty zany, but you need to show us the necessary connection between Aristotle's science and the act/potency distinction.

Anonymous said...

ozero91

"a mathematical description cannot cause anything."

Quite right. Our laws of physics describe the way things behave. If things behaved differently, our physical laws would be formulated differently. So when we say things behave in accordance with the laws of physics, it means things behave in the way our formulas describe, which is true, because the formulas are derived from the observed behavior. It doesn't mean the formulas dictate their behavior.

Anonymous said...

"Aristotle's "science" was pretty zany, but you need to show us the necessary connection between Aristotle's science and the act/potency distinction."

Aristotle's Physics was zany, and his Metaphysics was just as zany. Read the books.

ozero91 said...

"Quite right. Our laws of physics describe the way things behave. If things behaved differently, our physical laws would be formulated differently. So when we say things behave in accordance with the laws of physics, it means things behave in the way our formulas describe, which is true, because the formulas are derived from the observed behavior. It doesn't mean the formulas dictate their behavior."

Indeed, but then I'm not sure what you were trying to convey here:

"I say that because everything we see behaves according to the laws of nature, and no further explanation is needed for why things "move"."

"Everything" means matter and energy right? And we see that matter and energy "behave," or in other words they exhibit certain features and characteristics, such as momentum. When two things collide, we notice that their momentum does not seem to disappear, rather the momentum is conserved. This behavior can be summarized in a mathematical language. We call these the laws of nature. Now we might ask, why matter exhibits these behaviors. Your answer would seem to be "the laws of nature." But the laws of nature are just our summary/description of behavior of matter.

ozero91 said...

"Aristotle's Physics was zany, and his Metaphysics was just as zany. Read the books."

I'm interested in the zany metaphysics. Which books?

BeingItself said...

Aristotle used the same methods to come up with his Physics as he used to come up with his Metaphysics.

Yet these guys somehow think that method was sound for metaphysics yet failed for physics.

Why?

Making stuff up and then calling it a metaphysical principle does not make it true. The method is madness.

Anonymous said...

ozero91,

"But the laws of nature are just our summary/description of behavior of matter."

So, what kind of behavior is described by 'moving'? What exactly is a state of 'potency' or a state of 'actuality'? Do these things help us understand how things behave? Do they describe any kind of regularity that we can use to bring order to our world? in truth, that kind of thinking kept people in the dark for many centuries.

ozero91 said...

"So, what kind of behavior is described by 'moving'? What exactly is a state of 'potency' or a state of 'actuality'? Do these things help us understand how things behave? Do they describe any kind of regularity that we can use to bring order to our world? in truth, that kind of thinking kept people in the dark for many centuries."

"Motion" in the scholastic sense means "change." Does change play a role in our explanation of matter?

William said...

"
don't you think causality looks more like a matrix than a chain?
"

It could be either or neither I think, and depends on our point of view and level of description.

Anonymous said...

I made it to about the eighth comment and quickly remembered why I'm not around comboxes much anymore.

"...to the rest of us, grounded in reality, the arguments are miserable failures."

Ah, such cogency! No need to, you know, actually refute the argument, just throw out a line and watch the theists bits.

Same ol' same ol', rinse wash repeat. See y'all in another month or three.

ozero91 said...

Can change be explained by something that is also changing?

Anonymous said...

William,

To say that A causes B, which causes C, etc. is too simplistic to describe how things really happen. There are many causes for any given event. The A-T view makes it overly simple, while forming the basis of arguments like the First Way. That casts those arguments into doubt.

Papalinton said...
This comment has been removed by the author.
Papalinton said...

The Unmoved Mover simply defies the understanding of the conservation of energy. Nothing we observe within this universe comports to anything like an Unmoved Mover. It is an imaginative place-marker in the absence of evidence that has no analogical comparator known to us. Declaring an unmoved mover has the explanatory power of the ubiquitous 'just because'.

We can only work and build on what we know. We cannot build on anything we just don't know. This is the paradox of the unbounded nature of the act of imagination.

We have known about unicorns since antiquity. Millions of words over the course of recorded history has recounted the life and feats of unicorns. Few if any would imagine the reality of unicorns existing outside and beyond the stories told of them. Nevertheless unicorns are a universal cultural memeplex by which we can project, anthropomorphise and personify their existence. The God concept is every bit identical in nature to the imaginative projection of unicorns. One thing we do know, supported by much evidence, is that this is counterintuitive to our perceived understanding and that is why the importance of learning, knowledge and understanding can never be underestimated if we are to grow and mature as a species beyond the comfort of our base instinctual proclivities.

ozero91 said...

Conjoining an essence with an act of existence, etc.

William said...

im:

The scholastic proofs of God rely on scholastic premises and presuppositions. Those scholastic presuppositions have very little intuitive force today.

grodrigues said...

@im-skeptical:

"I understand that the god of Thomas is defined as pure actuality, but isn't it a rather convenient definition."

The "god of Thomas" is not defined as pure act.

"Movement is defined as transforming from potency to actuality, but something unmoved is also actuality. Those things don't agree with each other. It still seems like a contradiction to me."

First, "transforming" is an extremely poor choice of wording. Second, *everything* in existence is a composite of act and potency; only God is pure act so your second sentence is not very informative. Third, is what "seems to" you supposed to be an argument?

[to Doug:]

"Do you think that the empty vacuum of deep space is actuality?"

Maybe you should explain by your own words what you think Aristotle means by potency and act.

"Even if they [laws of nature] are not immutable, it would still be fair to say that everything acts in accordance with them."

If "law of nature" is a mere descriptive summary of how things behave, and this is what you repeat in another post below, then saying that "everything acts in accordance with them" is a vacuous claim that explains absolutely nothing.

[To William:]

"There are many causes for any given event. The A-T view makes it overly simple, while forming the basis of arguments like the First Way."

Your statement is literally false. Anyway, whether there exists one cause or many causes for "any given event" is irrelevant to the thrust of the argument.

@Doug:

"Moreover, even a hypothetically empty space has the potential to be filled, and so it stands in causal relations."

Just to preempt some possible misunderstanding; God also stands in causal relations. It is the fact that space-time, *if* it is a substance in the first place (which is not all obvious), is likewise a composite of act and potency.

ozero91 said...

"[To William:]

"There are many causes for any given event. The A-T view makes it overly simple, while forming the basis of arguments like the First Way."

Your statement is literally false. Anyway, whether there exists one cause or many causes for "any given event" is irrelevant to the thrust of the argument."

Yo grod, I think you got your quoting mixed up. I think it was im skeptical who said that. See March 10, 2013 8:01 PM.

Anyways, this article is lengthy, but interesting in terms of physics and phil of sci.

http://sdcojai.wordpress.com/263-2/

grodrigues said...

@ozero91:

"Yo grod, I think you got your quoting mixed up. I think it was im skeptical who said that."

Yes, I know. The post starts with @im-skeptical, the one I am addressing, and all quotes are from him. The [To William:] is just a marker signalling that the quote is from a different post, namely one addressed to William. Applogies if my typographical conventions were not clear.

Unknown said...

Papalinton, I didn't read the paper. I'm not going to do busy work. However, if you find something relevant there, feel free to reproduce it here.

I'm stunned that you think we should abandon propositional logic. This is wrong?

1. A is B.
2. B is C.
3. Therefore, A is C.

The citation you reproduce hardly suggests that propositional logic is wrong. Rather, it distinguishes between a valid argument and a sound argument. A valid argument is one in which the conclusion follows from its premises, whether those premises are true or false. A sound argument is one that is valid and has true premises. In either case, there's nothing wrong with the logic.

Your objection that the Unmoved Mover cannot be observed and verified like the laws of nature is based on a principle that was almost universally abandoned fifty years ago. "We should only believe something is true if it can be observed" is itself incapable of being observed. Hence, your objection is self-defeating.

Besides, even though we make improvements in our knowledge of the laws of nature, that doesn't mean the laws of nature become any different than they were before.

Unknown said...

By the way guys, when I say the laws of nature are movers, I don't mean to say that the mathematical equations act upon potentialities. Mathematical equations are abstract objects, and therefore do not stand in causal relations. Instead, I'm referring to the forces that the laws of nature describe, and these forces do stand in causal relations.

Unknown said...

im-skeptical,

You write, "The point is that there is no such thing as a chain of causality. But everything does behave according to natural law."

This simply doesn't follow from the examples you gave. In each of the examples, there is a cause-and-effect relationship. The acorn that was supposed to be actualized into an oak tree simply had its potentiality unactualized. Yet, this was because some other potentiality was actualized, e.g. the city planning, etc.

Anonymous said...

doug,

"This simply doesn't follow from the examples you gave. In each of the examples, there is a cause-and-effect relationship. The acorn that was supposed to be actualized into an oak tree simply had its potentiality unactualized. Yet, this was because some other potentiality was actualized, e.g. the city planning, etc."

True, everything in the example was a "potentiality was actualized", but what may not have come across very well is that there are numerous interdependencies, all acting together to achieve the outcome. In fact those interdependencies are so numerous, you can practically say that everything causes everything else. Even if you don't have a determinist view, you still can't deny that the state of the world at one moment has a significant bearing on what the state will be in the next moment. The bottom line of all this is that Aristotelian notions of causation are irrelevant to the real world. They are simply inadequate to describe how events come about.

Anonymous said...

grodrigues,

"It is precisely because God *is* pure act that He can be, and is the origin of the chain."

"The "god of Thomas" is not defined as pure act."

Uh ... right.

"Third, is what "seems to" you supposed to be an argument?"

I was raising a question ... trying to clarify my understanding about what appears to be a contradicition.

"saying that "everything acts in accordance with them" is a vacuous claim that explains absolutely nothing."

Clearly the point was lost on you.

"whether there exists one cause or many causes for "any given event" is irrelevant to the thrust of the argument."

I don't think so. The argument says that there has to be one cause at the beginning of it all. But that's based on an invalid notion of causality. I think it's very relevant.

Son of Ya'Kov said...

@im-skeptical

Don't BS it will piss off both the g-man and myself.

Your wrote:
>"I understand that the god of Thomas is defined as pure actuality, but isn't it a rather convenient definition."

He responded.
The "god of Thomas" is not defined as pure act.

God is not conveniently Ad Hoc defined as Purely Actual rather logically eliminating the possibility of an infinite essentially causal chain said causal chain must begin with something that is pure act which men take to be God.

That was obvious.

Unknown said...

im-skeptical, I think you have a misconception about Aristotle's worldview. According to Aristotle, there world is, in fact, an interacting whole. Thomas expresses this in his Fifth Way, when he talks about the unity of diverse objects exhibiting regularity. I still don't see how this has any bearing on why you reject AT-metaphysics. The whole world is interconnected, whether it's deterministic or indeterministic.

grodrigues said...

@im-skeptical:

BenYachov already kindly dispatched for me your first non-response.

"saying that "everything acts in accordance with them" is a vacuous claim that explains absolutely nothing.

Clearly the point was lost on you."

So would you kindly enlighten me what was your point? According to you, the laws of nature discovered by scientists are mere *descriptions* of the behavior of objects, so they are not real, existent things that can *explain* anything at all. So what is *your* point in invoking them?

"The argument says that there has to be one cause at the beginning of it all. But that's based on an invalid notion of causality. I think it's very relevant."

That is at best, a misleading way of stating the conclusion, at worst, simply wrong. But first things first. Since you "think it's very relevant", in the *context of the First Way*, what does Aquinas mean by cause? And since you think the Aristotelian notions of cause are "irrelevant to the real world", please define for us causality writ large that is "relevant" to the real world and then show how it is different from Aristotelian notions. Moreover, how does the putative fact that some (and I will even grant you all, although this is not true) events have more than one cause invalidates the argument?

note: just to help you a little, here is one hint of where you are going wrong. First, Aquinas speaks about *beings*, not events or states. States, are always a state of *something* and an event is always some change in one or various beings, so states and events are ontologically dependent and explained in terms of beings and the changes they undergo, and thus act and potency.

ozero91 said...

To chime in: If my understanding is correct, it is the goal of the First Way to prove that Pure Act exists. Do we know that "Pure Act" is God? No we do not, not from the First Way alone. I think Aquinas argues in other places for the conclusion that "Pure Act" is God.

Anonymous said...

Doug,

The Fifth Way says to me that natural law is the result of an intelligence which governs all things. I don't read anything in it that speaks of interconnectedness. We agree that there is regularity, and that is natural law - what physicists describe with mathematical equations. But Aristotle still had his four causes, and this potency and act, and they still don't fit well with observed reality, as I tried to show with the deep space example.

Now you say that natural law itself can be the mover that turns water to ice, rather than some physical entity that acts upon the water. Fine. My response to that is that this is indistinguishable from saying that natural law is responsible for everything. And if that's the case, then Aristotle's four causes are still irrelevant. And the First way simply boils down to: Everything behaves according to natural law, and God makes it all happen. That's not the conclusion of any argument - it's merely an assertion.

ozero91 said...
This comment has been removed by the author.
ozero91 said...

"And the First way simply boils down to: Everything behaves according to natural law, and God makes it all happen."

Please demonstrate this. Start with the original argument.

Just explain to us, what is the problem with the act-potency distinction? Do note that the act-potency distinction does not require that there is only on "actualizer" per potency. In other words, even if you need two or more distinct actual things to converge in order to move a potency to an act, you still have act and potency.

[Edited post b/c I misrepresented something.]

ozero91 said...

IIRC, the four causes apply to things, not events.

And remember, the act-potency distinction is an explanation of why change is possible at all.

Unknown said...

im-skeptical, my apologies for not being specific, but I actually meant Thomas's Fifth Way not in the Summa Theologica, but in the Summa Contra Gentiles. It's there he talks about diverse objects coming together under one order.

Now, the deep space analogy, even if cogent, doesn't undermine other examples where there are instrumental causes. When you throw a baseball in the air, sure, gravity is at work, but it would never go up in the air unless you were the efficient cause of the action.

ozero91 said...

Water has the potential to lose it's average internal kinetic energy. In other words, heat energy has the potential to move from warmer body to a colder body. But that's all it is, a potential. In order to actualize that potential, you need an actual colder body.

Heat energy also has the potential to move from a colder body to a hotter body. But in order to actualize this potential, you need work (in terms of physics).

Anonymous said...

Doug,

Before commenting further, I need to read some additional material from Thomas. I didn't know there was another Fifth Way, so let me check it out.

ozero91,

"In order to actualize that potential, you need an actual colder body."

That is the way I think Thomas saw it, but I have shown that it's not true.

Son of Ya'Kov said...

> I didn't know there was another Fifth Way, so let me check it out.

Just to be clear there is only one Fifth Way. Summa Contra Gentiles merely fleshes it out more then Summa Theologica. The later being a book designed to give a summery to beginning students of philosophy.

Anonymous said...

Doug,

I'm afraid I couldn't find what you were referring to. Which chapter is it in?

I did find this in Chapter 13:

"[13] Furthermore, that it is impossible for the abovementioned infinites to be moved in a finite time Aristotle proves as follows. The mover and the thing moved must exist simultaneously. This Aristotle proves by induction in the various species of motion. But bodies cannot be simultaneous except through continuity or contiguity. Now, since, as has been proved, all the aforementioned movers and things moved are bodies, they must constitute by continuity or contiguity a sort of single mobile. In this way, one infinite is moved in a finite time. This is impossible, as is proved in the Physics [VII, 1].

[14] The second argument proving the same conclusion is the following. In an ordered series of movers and things moved (this is a series in which one is moved by another according to an order), it is necessarily the fact that, when the first mover is removed or ceases to move, no other mover will move or be moved. For the first mover is the cause of motion for all the others. But, if there are movers and things moved following an order to infinity, there will be no first mover, but all would be as intermediate movers. Therefore, none of the others will be able to be moved, and thus nothing in the world will be moved.

[15] The third proof comes to the same conclusion, except that, by beginning with the superior, it has a reversed order. It is as follows. That which moves as an instrumental cause cannot move unless there be a principal moving cause. But, if we proceed to infinity among movers and things moved, all movers will be as instrumental causes, because they will be moved movers and there will be nothing as a principal mover. Therefore, nothing will be moved."

So this tends to confirm the idea that Thomas views causality as a chain, and also discount the notion of natural law as being an instrumental cause.

Papalinton said...

Doug Benscoter
"I'm stunned that you think we should abandon propositional logic. This is wrong?
1. A is B.
2. B is C.
3. Therefore, A is C.
The citation you reproduce hardly suggests that propositional logic is wrong. "


You seem to have conveniently omitted my following reasons for dispensing propositional logic if it continues to produce a false positive. And you seem to have missed the vital conditional aspect of propositional logic; to wit: " ... whose truth-value depends entirely on the truth-values of the simpler statements making them up, and in which it is ASSUMED that every statement is either true or false and not both.". I will restate what I wrote:

"Yes we should abandon them IF [my bolding and capitals] they have been found less than sufficient to prosecute an argument, because clearly propositional logic is only as good as, and relies utterly dependent and captive to the veridicality of the hypothesized premise; and if that initial premise is false, propositional logic, itself being simply a process, will not flush out the falsity of the premise, but carry it through to an 'apparent truth' of its conclusion.

The process of propositional logic cannot by and of itself distinguish between false or true statements. These can only be assumed. Their veridicality must rely on other sources of knowledge and information for substantiation external to the proposition. It is precisely this ambivalent characteristic of propositional logic that theists and philosophers of religion have fought so hard for and have been clinging so dearly onto [and banging on about] to argue their case for the apparent existence of an Unmoved Mover, the existence of a God entity that inhabits a supernatural realm.

The Kalam is a superb example of the looseness of propositional logic. Can you recall the totality of WLC's argument in support of the Kalam is? Here it is:

"William Lane Craig argues that the first premise is strongly supported by intuition and experience. He asserts that it is "intuitively obvious", based on the "metaphysical intuition that something cannot come into being from nothing" Wiki

Intuition? Experience? And you think Craig's rationalizing is a sufficient candidate for proposition logic based on intuition and personal experience?

Sorry Doug, you really need to pull the other leg if theological conceptions are going to continue underwriting your philosophical argument. The Unmoved Mover is a euphemism for the Christian god, No amount of denial, obfuscation, or obscurantism will allay that conclusion.

I simply cannot go past Doctor Logic's apt description of the relevance of theologically-derived philosophy. It is increasingly becoming a joke and a somewhat shabby and tired one.

Unknown said...

Papalinton, Aristotle wasn't a Christian. It was he who first coined the term, "Unmoved Mover." Do you understand the difference between the kalam argument and the argument from motion?

Also, I didn't omit anything you wrote. Question: do you know the difference between a valid argument and a sound one? Logic doesn't need to establish a proposition's truth-value in order to be needed. Your denial of logic presupposes it, and so is self-defeating.

Unknown said...

im-skeptical, here's the relevant passage. It's at the end of Chapter 13:

“Contrary and discordant things cannot, always or for the most part, be parts of one order except under someone's government, which enables all and each to tend to a definite end. But in the world we find that things of diverse natures come together under one order, and this not rarely or by chance, but always or for the most part. There must therefore be some being by whose providence the world is governed. This we call God.”

He doesn't call it the Fifth Way, but it most resembles that argument.

Anonymous said...

Doug,

That passage does seem to support the Fifth Way, but it says nothing about the interconnectedness of causality. The earlier passage that I cited specifically refers to "an ordered series of movers and things moved". That's the way Aristotle saw things, I believe, and Thomas too. And that's the view that I have objected to as being overly simplistic, pre-scientific, and not representative of the real world.

grodrigues said...

@im-skeptical:

[to ozero91:]

"In order to actualize that potential, you need an actual colder body.

That is the way I think Thomas saw it, but I have shown that it's not true."

Although ozero91's formulation is not correct, your example of water freezing in deep space shows no such thing. Water is composed of parts -- water molecules -- and it is the interaction of these parts, the exchange of energy between them, and energy radiating to the environment that explains the water freezing. Nothing in here contradicts the metaphysical Aristotelian ideas, in the same way as animals (e.g. human beings) being self-movers, meaning having the capacity of locomotion of themselves without being moved by anything else, shows otherwise.

[to Doug:]

"That passage does seem to support the Fifth Way, but it says nothing about the interconnectedness of causality."

Would you care to explain in what this "interconnectedness of causality" is exactly and how it defeats the Thomistic metaphysical ideas of act and potency? And just to be sure we are on the same page, what is the distinction between act and potency and why exactly was Aristotle led to posit it? The reason why I am asking this, is because of statements like,

"So, what kind of behavior is described by 'moving'? What exactly is a state of 'potency' or a state of 'actuality'? Do these things help us understand how things behave? Do they describe any kind of regularity that we can use to bring order to our world? in truth, that kind of thinking kept people in the dark for many centuries."

that just betray ignorance and confusion, which then, in an ironical twist, is imputed to Aristotle and his progenie. But maybe I am just misreading you, thus my question. One more for the road:

"True, everything in the example was a "potentiality was actualized", but what may not have come across very well is that there are numerous interdependencies, all acting together to achieve the outcome. In fact those interdependencies are so numerous, you can practically say that everything causes everything else. Even if you don't have a determinist view, you still can't deny that the state of the world at one moment has a significant bearing on what the state will be in the next moment."

No one denies that the past influences the future. But saying that "those interdependencies are so numerous, you can practically say that everything causes everything else" is of course literally false. So what can you possibly mean by it? I submit that whatever you mean by it, it can *always* be analyzed into the Aristotelian categories of act and potency and reduction of potency to act and the principle of causality.

Son of Ya'Kov said...

to grodrigues

>[to Doug:]

>"That passage does seem to support the Fifth Way, but it says nothing about the interconnectedness of causality."

Don't you mean [to im-skeptical]

Cheers bro!:-)

ozero91 said...

@ben, I was confused too, but it seems grod has his own quoting system.

And if my formulation is not correct, well I guess I should go back to the drawing board... or hit the books.

Also, why is potency said to be "reduced" to act? Is potency considered to be "higher" than act in some way?

Papalinton said...

Doug
'Papalinton, Aristotle wasn't a Christian. It was he who first coined the term, "Unmoved Mover."

Correct. I am talking of the contemporary understanding, talking of your and my cultural milieu. Aquinas appropriated Aristotle's unmoved mover, And Aristotle being the Pagan he was his unmoved mover was not the christian god to be sure, by quite a few centuries. But it was Aquinas who conjoined the two disparate concepts of Aristotle's unmoved mover and the christian god. And it is A-T philosophy that is being promulgated by today's religionists. It is Aquinas's conception of the unmoved mover that I speak of today.

From your comment, it seems I do have to spell it out, without assuming that today's theists can follow the context and underlying nuance of the conversation. So I hope this clears the matter a little with you.

"Do you understand the difference between the kalam argument and the argument from motion?"

Once again you prevaricate. The Kalam was selected as a perfect example of the shortcomings of the process of propositional logic in arguing for the notion of a prime mover or uncaused caused to get a ball rolling. But the concept of Aquinas's First Way of positing an argument from motion is equally flawed. It cannot distinguish whether premise A or B are indeed false. They are assumed to be true, as WLC says of the Kalam, by dint of intuition and personal revelation [experience] only.

"Summary of the First Way
The argument of the unmoved mover, or ex motu, tries to explain that God must be the cause of motion in the universe. It is therefore a form of the cosmological argument. It employs Aristotle's dichotomy of potentiality and actuality. It goes thus:
Some things are in motion.
A thing cannot, in the same respect and in the same way, move itself: it requires a mover.
An infinite regress of movers is impossible.
Therefore, there is an unmoved mover from whom all motion proceeds.
This mover is what we call God.
NB: it must be kept in mind that throughout Aquinas and Aristotle, 'motion' has a much wider meaning than the modern sense of the word: it is a very wide synonym for 'change', it does not only refer to change of position in space as the modern term does."
Found HERE.

The elephant in the room in Aquinas's proposition is, An infinite regress of movers is impossible. Says who? Evidence is required [and a little more than intuition, or personal revelation or on undisciplined reliance on propositional logic itself [which you must remember, is a process incapable of distinguishing false or true statements and can facultatively proceed to a supposedly reasoned but ultimately false conclusion] Thereby, in making this unfounded and unsubstantiated proposition, Aquinas tenuously posits that the christian god is the only logical conclusion. This old trope of course is pretty much a hairy chestnut that is taking some time to work through by christian apologists but in the end it will be found to be little more than a factoid, inconsistent with our increasing knowledge base and understanding of the nature of the universe. Time here is not of the essence. Simply the natural devolution of religious thinking as a measure of discerning truth.


Anonymous said...

grodrigues,

"Nothing in here contradicts the metaphysical Aristotelian ideas, in the same way as animals (e.g. human beings) being self-movers, meaning having the capacity of locomotion of themselves without being moved by anything else, shows otherwise."

Except the first way says things don't move themselves, and I'm sure that would apply to a jar of water.

"Would you care to explain in what this "interconnectedness of causality" is exactly and how it defeats the Thomistic metaphysical ideas of act and potency?"

Would you care to read what I already wrote?

"what is the distinction between act and potency and why exactly was Aristotle led to posit it?"

This in response to me asking what those things really are and how we can use them to describe what happens in the real world. Why don't you tell me?

"that just betray ignorance and confusion"

You could help clear up my confusion.

"you can practically say that everything causes everything else" is of course literally false."

How so? No, nevermind. All you want to do is spout about how wrong I am. Remember when I argued that you can't get to infinity by counting? What did you have to say about it? That I didn't know what I was talking about. Now in this thread, Doug points out that Aquinas made essentially the very same argument. But you didn't jump in and say that Aquinas was an ignorant fool. Maybe you are just unwilling to even try to understand anything I say, or consider that there might be any truth at all in it. You'd rather just assume that it's all wrong and throw stones at it. So be it.

grodrigues said...

@ozero91:

"And if my formulation is not correct, well I guess I should go back to the drawing board... or hit the books."

The formulation is incorrect (or misleading) insofar as the specific example you gave, not as a general principle, as a body can get colder simply by radiating energy. Of course, you could say the external environment is the colder body, but this is bound to be confusing, as the external environment (empty space in im-skeptical's example) is not a body in the usual meaning of the term.

note: and sorry about my quoting conventions. Instead of [to X:] I better write something like [addressing X:] or something like it.

grodrigues said...

@im-skeptical:

"Except the first way says things don't move themselves, and I'm sure that would apply to a jar of water."

Methinks you are confusing the First Way with the principle of causality. As far the example I gave with animals being self-movers (or your example with water, the idea is the same) it is simply the fact that animals are composed of parts and it is the different parts of the animal that move one another, and together give the animal locomotion.

"This in response to me asking what those things really are and how we can use them to describe what happens in the real world. Why don't you tell me?"

So if you do not know what they are, then why do you write such things as "in truth, that kind of thinking kept people in the dark for many centuries"? I am just asking what is the basis for such pronouncements. So you have none, OK.

"Remember when I argued that you can't get to infinity by counting? What did you have to say about it?"

I am pretty sure I never argued that, but feel free to quote me and if I am wrong, well, my sincere apologies are in order then.

"you can practically say that everything causes everything else" is of course literally false.

How so?"

Simply because every cause as a typical effect or specific range of effects, for otherwise anything would follow from everything, which is blatantly false. Do you have another meaning in mind? Maybe you want to say that for some effect there is a large number of contributing factors and causes. But this in no way invalidates what Aquinas is getting at, because either this large number of contributing factors can either be analyzed in more basic causes (e.g. a typical example is macroscopic phenomena like pressure and temperature being analyzed in terms of large number of basic causal chains) or even when there is no such analysis, we can simply focus attention on *one* causal chain and run the argument to get at the unmoved mover -- and remember that the causal chains we are interested are essentially ordered causal chains.

"But you didn't jump in and say that Aquinas was an ignorant fool. Maybe you are just unwilling to even try to understand anything I say, or consider that there might be any truth at all in it. You'd rather just assume that it's all wrong and throw stones at it. So be it."

So let me see if I understand you. I did not call you an "ignorant fool" anywhere in this thread, or any other name for that matter -- should I have? Rather, I asked you for clarification of what you mean by a certain statement, e.g. "interconnectedness of causality", because in my understanding of what you mean by it, it is either irrelevant or literally false. I asked what you understand by act and potency to make sure we are on the same page, since some sentences of yours betray ignorance and confusion, but that it could be that I was misreading you. In fact, I have made these and similar questions in my other posts addressed to you; and you have not responded. And all this is "unwilling[ness] to understand anything" you say or "throw[ing] stones"?

If you do not want to clarify or answer my questions, you are in your every right -- you have done it already and you have not heard a peep from me. But this childish pouting? Give me a break.

Anonymous said...

"you have not heard a peep from me. But this childish pouting? Give me a break."

Sure thing.

Unknown said...

im-skpetical,

"Under one order" seems to suggest interconnectedness. In any case, how does interconnectedness undermine Aristotelian metaphysics. I still don't understand your object. As I said, if you throw a baseball in the air, then you're an instrumental cause. If you're removed from the picture, then the laws of nature won't actualize such a potentiality.

Unknown said...

Papalinton, I don't see any serious interaction with what I've written on your part. The fact remains that you presuppose logic in your rejection of it. Moreover, I've already provided arguments in favor of the First Way's premises. If you don't want to respond to them, then that's fine, but you haven't attempted to undermine my defense of it. Be well.

Anonymous said...

Doug,

"Under one order" refers to single governing entity as the orchestrator of it all. I don't think he's talking about interconnected causation. But there is the discussion of "an ordered series of movers and things moved" as part of his view of causation. This is contrary to the view that I have presented. The First Way postulates a chain of causation that goes back to the first mover. My contention is that in reality, there is no chain of causation. So the argument falls apart. Of course, that's debatable.

Unknown said...

The Fifth Way aside, wouldn't you view yourself as an instrumental cause in throwing a baseball in the air?

By the way, I appreciate the civilized debate. You seem to have a good head on your shoulders.

Anonymous said...

Doug,

If we take a limited view of causation, yes, you could say something like "I am the instrumental cause in the movement of an object". But the First Way does not purport to take such a limited view. It goes all the way to God. So we need to take a broader view of causation - and that entails the recognition of the interconnectedness of things.

William said...

With regard to context determining a matrix versus a chain of causation,or whatever, causation, I recommend Johnathan Schaffer:

http://www.jonathanschaffer.org/causalcontext.pdf

Doctor Logic said...

I agree with William.

We can describe the universe as a series of states that are located or indexed in space and time. Change, then, would be an abstraction: a difference in states across a boundary of space or time.

Regularities in the map of states are our laws and forces. According to such regularities on the map, some later states are fully or partially determined by prior states. From this, we get a technical notion of causation. Prior states "cause" later ones in the sense that, inductively, we expect the rules to apply in the future just like they did in the past. The future states (or next states in a purely spatial series) are fixed (or partially fixed) by past states. Laws are like axioms of the system. Only certain futures are consistent with the past.

If we follow these states all the way back into the past (and the series terminates), there will be an initial state that does not depend on anything before it.

In this picture, act and change are not fundamental. The states are fundamental, and the differences between them are just abstractions. They're the map. If there's an initial state, then searching for an initial act is nonsense because there's no change if there's no prior state to change from.

There's nothing inconsistent about this state-oriented picture.

Doctor Logic said...

Naively, we might think we can paraphrase this description so that the acts/changes are fundamental and the states are inferred. This is not what physicists do, but for argument's sake, let's suppose we can do it. If we turn the clock back to the beginning of the universe, we'll see something perverse. An act creates the first state, not just out of a vacuum, but in a nonexistent coordinate system. There's no "before" in which to create the first state. For people judging this contest on the basis of what is intuitive, can you think of an act without any context whatsoever? Intuitively, contexts are as vital to acts as prior causes are to effects. Not that I'm recommending you stick with intuition...

Asking what caused the universe is equally nonsensical unless you embed the universe inside another space/universe. If you do that, then we're back where we were to start with.

Doctor Logic said...

A few words about potency. Just because I can conceive of a universe that is not a sphere does not imply that the spherical universe was merely contingent, and that the universe might have been, say, a cube. Conceivability and potentiality have different meanings. I can conceive of having the potential to zap my cat with laser vision, but I don't actually have that potential.

If there is a beginning to the universe, then there are going to be inexplicable, brute facts that have no cause. Any answer to the question "why is there something rather than nothing?" is going to be another something.

In the states-oriented physicalist picture, the physical laws and the initial state of the universe are brute facts that have no explanation. If you invoke God, then the reason why God made his particular choices for our universe is a brute fact that cannot be explained (not even by God). Sagan was right. What's the point of postulating an invisible, all powerful being whose brute facts are equivalent to the brute facts of our visible universe?

William said...

Strangely perhaps, there is a parallelism and continuity in Dr Logic's physicalist metaphysics and the Aquinean essentialism project. Both start with both empircal facts and add to that a large number of empirically unwarranted assumptions related to a historical body of theory which is likely wrong in many respects. In Aquinas' case, the act/potency assumptions were based, not just on the metaphysics, but on the assuptions of the physics of Aristotle's time. Because of the facile nature of metaphysics, the Aquinian act/potency theories still are useful in metaphysics, even if they have lost touch with current physics.

In the same way, Dr Logic's physicalism is based on 20th century theories which are already changing and will probably not survive another hundred years, except as the branch of metaphysics called physicalism today. Philosophy is often far more conservative than its children the sciences.

One difference between physicalism and act/potency/essence theory is that, because it became a matrix for theology in the Roman Catholic Church, Aquinean theory will greatly outlive 20th century physicalism, just as human rights metaphysics will last at least as long as the nations which enshrined it in their inception.

I admit I kind of like that.

Son of Ya'Kov said...

@DL

You are obsessed with the Kalam Cosmological Argument and you live under the delusion that all Cosmological Arguments presuppose the Universe had a beginning & are but a variation of that.

What part of "The First Way and Aquinas does not presuppose we can philosophically or scientifically prove the Universe had a beginning" do you not understand?

Son of Ya'Kov said...

>Strangely perhaps, there is a parallelism and continuity in Dr Logic's physicalist metaphysics and the Aquinean essentialism project.

Except based on my dealings with DL he thinks materialist physicalism is a scientific view not a metaphysical one.

The man uncritically holds to Positivism the way many simple believers hold to Fideism.

It's madness.

Son of Ya'Kov said...

>A few words about potency. Just because I can conceive of a universe that is not a sphere does not imply that the spherical universe was merely contingent, and that the universe might have been, say, a cube. Conceivability and potentiality have different meanings. I can conceive of having the potential to zap my cat with laser vision, but I don't actually have that potential.

Classic Hunmean error! Equivocating between conceiving vs imagining it. They are not the same.

Papalinton said...

Doug
It's serious enough. From the content and the rationale of your various commentary It seems your arguments are pretty much predicated on an invariable and inflexible Thomistic perspective, just as it has been handed down from Aquinas, an unrevised version protected solely by apologetical reluctance to meddle with what was once regarded as the ultimate defeater defeater, in the absence of the sheer quantum of scientific and natural knowledge that we are now privy to. It is a reluctance, spurred on by virtue of tradition, to change a version that has yet to account or respond to the many refutations and shortcomings identified since scientific inquiry rightly divested itself of its religious underpinnings during the Enlightenment period. A-T philosophy, by contrast, continues to promote and propagandise outmoded and outdated philosophical ideas that remain inextricably wedded to christian theology as its primal knowledge base.

Your reluctance to review Hennessy's paper, a robust and telling critique of Aquinas's First Way, is symptomatic of that reluctance about which I speak. In the absence of serious engagement to address the challenges, Aquinas buffs continue to traffic in the same old, same old. But it is a somewhat pointless and unproductive pursuit as contemporary philosophical trends have confirmed, with A-T philosophers largely corralled into academic enclaves by self-segregation.

Doctor Logic said...

BenYachov,

What part of "The First Way and Aquinas does not presuppose we can philosophically or scientifically prove the Universe had a beginning" do you not understand?

It doesn't matter because the logic is still the same. To discuss the creation or sustaining of the universe, you'll have to postulate an unnecessary external space or context in which that happens. This postulated external space exists for solely theological purposes, and it just moves the mystery one step further away. It has no explanatory power, only theological power.

Instead of saying the universe is a brute fact, you say your universe-making sustaining power is a brute fact.

Whatever is in motion is put in motion by another.

Not necessarily true. Radioactive decay is a fine example of this. There is change, but there is no cause for that change. Even if you don't believe in quantum randomness, there's nothing inconsistent about quantum randomness. Causality and familiar Newtonian mechanics work just fine as long as some things are effects of prior causes. If some things occur without a cause, there's no problem or contradiction.

ozero91 said...

Quantum tunneling via quantum fluctuations?

Son of Ya'Kov said...

@DL
>It doesn't matter because the logic is still the same.

So far your history of self-contradiction & backpedding hasn't exactly filled me with confidence in regards to your so called "logic" or lack their off.

>To discuss the creation or sustaining of the universe, you'll have to postulate an unnecessary external space or context in which that happens.

Only if in your kneejerk belief in Positivism you keep insisting it's a scientific question not a philosophical metaphysical one. Which youd still don't get. One merely has to know the difference between an accidental causal series(which according to Aquinas need not have a formal beginning) vs an essential series which logically needs a first cause to explain it's potency being reduced to actuality.

>This postulated external space exists for solely theological purposes, and it just moves the mystery one step further away. It has no explanatory power, only theological power.

Again you are casting it as an argument from physics which is both begging the question and a catagory mistake.

>Instead of saying the universe is a brute fact, you say your universe-making sustaining power is a brute fact.

Rather the brute fact is in an essential causal chain there must be a First Cause that is pure act or can you explain the motion of a proverbial caboose via attaching suscessive unpowered boxcars to pull it without making reference to a locomotive?

>Not necessarily true. Radioactive decay is a fine example of this. There is change, but there is no cause for that change.

Atheism-of-the-gaps! You are no better then an ID advocate who would claim god-did-it at this point. By nature we cannot directly observe the sub-atomic world without changing said world by observing it so there is no reason to postulate causality ceases at that level or to additionally postulate a supernatural explaination as to what is likely a non-local hidden natural phenomena.

>Even if you don't believe in quantum randomness, there's nothing inconsistent about quantum randomness. Causality and familiar Newtonian mechanics work just fine as long as some things are effects of prior causes. If some things occur without a cause, there's no problem or contradiction.

Except a cause from nothing is a contradiction as GEM Anscombe showed. You cannot assert a cause from nothing because there is nothing to cause anything. You are no better then someone postulating magic at this point.

The irony being you supose to be the scientific "rationalist" who rejects magic.

Ironic indeed.

ozero91 said...

I think it could be argued that decay involves instrumental causes. A proton or alpha particle derives it's causal power via particles of like charges.

Doctor Logic said...

BenYachov,

Lots of bluster, but you're being hypocritical.

First of all, I'm not making a scientific inference to a philosophical conclusion. I'm just pointing out that the current, standard interpretation of quantum mechanics is a counterexample to your philosophy. All I have to do is show that your philosophical claim isn't necessary, and that it is instead mere speculation (motivated by theological goals, at that).

By nature we cannot directly observe the sub-atomic world without changing said world by observing it so there is no reason to postulate causality ceases at that level or to additionally postulate a supernatural explaination as to what is likely a non-local hidden natural phenomena.

Incorrect, but let's not get sidetracked by your misunderstanding of QM.

You're being hypocritical. You implicitly acknowledge my argument, but you are unwilling to turn it back on your own.

You're saying that it is speculation on my part to assume that apparently random quantum phenomena are truly lapses of causality. Therefore, you say, if I were to argue that radioactive decay is an effect without a cause, and use this to try to prove a necessity, I would be in error. AND I AGREE!

If we can come up with a consistent counterexample to a claim of necessity, then the claim of necessity is demolished.

Now apply this same principle back onto Aquinas! I'm NOT saying that QM/radioactive decay is certainly an example of an effect without a cause (even though that's the way scientists treat it). What I am saying is that there is no contradiction if QM is truly random and if there really are some effects without causation. It is speculation to claim there is definitely determinism or definitely randomness.

Thus, likewise, it is speculation to demand that every event have a cause or a "mover". And any philosophical argument based on speculation renders its conclusions merely speculative.

A more philosophical argument looks like the following. If there are cause-effect relationships that necessitate final states given initial states, there will be some things that are not necessitated by the chain of causality. Whether those things are initial conditions, physical laws, the mind and preferences of God, or whatever. Any system like this is under-determined. Things do not explain themselves. Once we have used up all our causal explanations, we'll be left with some unexplained facts.

The initial state of the universe and its physical laws (and possibly the outcomes of random events like decays, if they are random) can just as well be those unexplained events as some external God. The God hypothesis doesn't do any explaining. You can't predict the physical constants from your theology, which means that your theology isn't explaining those things.

ozero91 said...

Per se vs per accidens, principle vs instrumental etc.

ozero91 said...

"Whatever is in motion is put in motion by another."

Is this premise identical to/interchangeable with "Whatever is reduced from potency to act is reduced from potency to act by something already actual."?

grodrigues said...
This comment has been removed by the author.
grodrigues said...

@ozero91:

"Whatever is in motion is put in motion by another.

Is this premise identical to/interchangeable with "Whatever is reduced from potency to act is reduced from potency to act by something already actual."?"

The second is the precise formulation of the principle of causality, from which the first follows since nothing in potency can reduce itself to act for it would need to be already in act to reduce itself to act, but this would entail that it would be both in act and potency (in some, the same, respect). Contradiction.

ozero91 said...

I think this is relevant:

http://faculty.fordham.edu/klima/SMLM/PSMLM10/PSMLM10.pdf

Ctrl-F for: 4. Natural motion

Son of Ya'Kov said...

>First of all, I'm not making a scientific inference to a philosophical conclusion.

I am still not convinced you actually know the difference.

>I'm just pointing out that the current, standard interpretation of quantum mechanics is a counterexample to your philosophy.

What type of "standard interpretation"? If it's an "interpretation" then isn't it a philosophical modelling?
What is the philosophical justification for this model?

>All I have to do is show that your philosophical claim isn't necessary, and that it is instead mere speculation (motivated by theological goals, at that).

No you have to make philosophical arguments for your interpretation & for the metaphysics that underly it. For example if your philosophical interpretation involved anti-realism (Aquinas and Aristotle require moderate realism) you must argue for anti-realism. You must polemic moderate realism. Sure I could pull out Parmedies or Heraclides to answer Aristotle but I would actually have to make an argument for their metaphysics and polemic Aristotle's.

You have not made an actual argument against the First Way you just stated the conclusions of another model.
Now do be fair I do that for breverty's sake but you have to show us how the AT metaphysics are wrong and the competeing metaphysics are correct via philosophical argument.

I'd like to see that for once.

>Incorrect, but let's not get sidetracked by your misunderstanding of QM.

>You're being hypocritical. You implicitly acknowledge my argument, but you are unwilling to turn it back on your own.

My argument is based on philosophy not science. I reject ID what part of that do you not understand? You see all religion threw the lens of ID Fiat creationism & you can't deal with anything else.

>You're saying that it is speculation on my part to assume that apparently random quantum phenomena are truly lapses of causality. Therefore, you say, if I were to argue that radioactive decay is an effect without a cause, and use this to try to prove a necessity, I would be in error. AND I AGREE!

So you are backpedaling because you realize I am not an ID advocates. When you learn Thomism is not ID with Rosary
beads you might actually one day venture a coherent argument that we can answer or stuggle with.

>Now apply this same principle back onto Aquinas! I'm NOT saying that QM/radioactive decay is certainly an example of an effect without a cause (even though that's the way scientists treat it). What I am saying is that there is no contradiction if QM is truly random and if there really are some effects without causation. It is speculation to claim there is definitely determinism or definitely randomness.

More back peddling.

I don't see what randomness has to do with anything since I can concieve of an atheist universe that is governed by quantum super determinism & thus is not random or we would simply say randomness is merely spontaneous causes that are unforseen. Aristotle wasn't a determinist and Aquinas had no problem with random events.

You are all over the place.

Son of Ya'Kov said...

>Thus, likewise, it is speculation to demand that every event have a cause or a "mover". And any philosophical argument based on speculation renders its conclusions merely speculative.

Rather I still have no philosophical argument as to why ex nihilo nihil fit is wrong only speculation that it could be. Yes your argument is no better than the creationist who says "Well God could have put dinosaur bones in the Earth to test our faith or create the light between Andromedia and the Milky Way so it's just speculation the world is older than 10000 years.

>A more philosophical argument looks like the following. If there are cause-effect relationships that necessitate final states given initial states, there will be some things that are not necessitated by the chain of causality.

That makes no sense it's a contradiction. You are arguing there is an effect without a cause since if there are cause-effect relationships regardless of their nature(quantum physics, Enstein's physics, Newton,etc) then they are part of a chain of causality.

>Whether those things are initial conditions, physical laws, the mind and preferences of God, or whatever. Any system like this is under-determined. Things do not explain themselves. Once we have used up all our causal explanations, we'll be left with some unexplained facts.

So like the Theist you embrace mystery. Well that's something if only you embraced philosophy and philosophical coherent & ditch the one size fits all anti-ID polemics we might have an actual argument.

>You can't predict the physical constants from your theology, which means that your theology isn't explaining those things.

I never said nor has any Thomist said that you could. In addition to confusing science with philosophy & other forms of latent Positivism you are now channeling the meme Theology=philosophy.

Really DL would it kill ya to learn some philosophy?

Just a little?

Son of Ya'Kov said...

Thank you grodrigues PAY ATTENTION DL! that is the correct formulation of the first way.

Granted my persons are confused by the first way because they strictly think of motion in Newtonian terms of physical movement from point A to B and not in metaphysical terms.

Parmedies=motion is unreal.

Heraclides=only motion is real stasis is unreal.

Plato=in a sense both Heralides and Parmedies are right.

Aristole=both motion and stasis are real in terms of Actuality and Potency.

Carry on I have work to do.

Doctor Logic said...

BenYachov,

All that arrogance and bluster from you just so you could confirm exactly what I said?!

Your philosophy makes unnecessary, speculative metaphysical assumptions about reality.

One can accept the reality of states of affairs, of motion, and causality without demanding that everything (without exception) fit under a simplistic and childish view of causality.

An adequate model of the reality we experience is like the following. The state of the system at t2 is mostly determined by the prior state of the system at t1. Those determining factors are physical laws or other regularities.

If some part of the current state is under-determined by the past, that doesn't call for anti-realism about causality, physical laws or the states themselves. It merely highlights the fact that blanket statements that every aspect of every final state has a cause are unnecessary, overreaching assumptions and speculations.

AT philosophy is speculation (with dubious semantics) for the purpose of safeguarding theistic fantasies. This isn't about realism or anti-realism.

And, no, what I said is not a contradiction. If there is a rule that necessitates subsequent states given prior states, then there will be some things (e.g., initial states and the rules of causality/physics) that are not necessitated under those same causal rules.

Speculation is fine, but don't pretend that any valid argument stemming from it has any soundness.

Son of Ya'Kov said...

So now folks philosophy doesn't just equal theology it equals "speculation" it can't be used to find truth.

Positivism on crack!

Dennett in a rare moment of non-Gnu philosophical insight said There is no such thing as philosophy-free science; there is only science whose philosophical baggage is taken on board without examination.

That is even more so for those who don't know any philosophy & refuse to learn.

DL I repeat like Dawkins you are not fit to polemic anything above young Earth fiat Creationism & or ID and Theistic Personalism.

That is you sole Atheist Skill set.

ozero91 said...

I guess nobody will check out my link, so I'll just post the relevant section here.

"Natural motion: If Newton is closer to the Aristotelians than is often supposed, so too are
the Aristotelians (or at least Aristotle and Aquinas) closer to Newton than is often supposed.
As James A. Weisheipl has shown, the idea that Aristotle and Aquinas held that no object can
continue its local motion unless some mover is continuously conjoined to it is something of
an urban legend.12 To be sure, this was the view of Averroes and of some Scholastics, but not
of Aristotle himself or of St. Thomas. On the contrary, their view was that a body will of
itself tend to move toward its natural place by virtue of its form. That which generates the
object and thus imparts its form to it can be said thereby to impart motion to it, but neither this
generator nor anything else need remain conjoined to the object as a mover after this
generation occurs. Aquinas comments:

[Aristotle] says, therefore, that what has been said is manifested by the fact that natural bodies
are not borne upward and downward as though moved by some external agent.
By this is to be understood that he rejects an external mover which would move these bodies
per se after they obtained their specific form. For light things are indeed moved upward, and
heavy bodies downward, by the generator inasmuch as it gives them the form upon which
such motion follows... However, some have claimed that after bodies of this kind have received their form, they need to be moved per se by something extrinsic. It is this claim that
the Philosopher rejects here."

http://faculty.fordham.edu/klima/SMLM/PSMLM10/PSMLM10.pdf

ozero91 said...

Think of all the protons in a nucleus. That is some serious repulsion, but not enough to break out of the nucleus. What allows an alpha particle to escape? Quantum tunneling.

Anonymous said...

DL,

Do you consider what you’re arguing is categorically different than trying to convince a creationist that ‘kinds’ of animals is not a useful concept to describe nature? Because that is what Thomism looks like to me: pseudoscience. It’s no wonder they aren’t taken seriously by scientists and the only retort is that they don’t understand it and if they you understood it, they’d agree with it, which get’s a prompt Courtier’s reply. Is this your impression as well?

ozero91 said...

"It’s no wonder they aren’t taken seriously by scientists and the only retort is that they don’t understand it and if they you understood it, they’d agree with it, which get’s a prompt Courtier’s reply."

So who are these scientists?

Son of Ya'Kov said...

Love the anti-philosophy fundamentalism & Positivism that plagues the brain dead gnus who think mere denial of gods automatically confers education and enlightenment & coherent rational argument.

BTW dguller who posts over at Feser's blog understands Thomism. He's still an Atheist & he's corrected other Atheists on the net for their misrepresentations and misunderstandings of AT metaphysics only to be abused by pretend to know it all's like the usual suspect here.

What is it about Gnus that makes them so butthurt over simply learning a topic before venturing a critique of it?

Also what makes them shout Courier's reply when their ignorance is exposed?

Lazy people.

ozero91 said...

Actually, I think it's more likely that modern scientists simply do not know the ins and outs of past metaphysical systems. They've just unwittingly inherited the metaphysics of the moderns, such as Descartes, etc.

ozero91 said...

Also, wasn't it Heisenberg who formulated a link between statistical quantum mechanics and the act-potency distinction?

Son of Ya'Kov said...

Yes, yes it was.

http://edwardfeser.blogspot.com/2009/09/heisenberg-on-act-and-potency.html

Cheers ozero91.

Anonymous said...

Ozero,
Correct me if I’m wrong, but isn’t Thomism not generally accepted among scientists? The usual suspects have commented negatively towards Thomism and I surveyed a couple blogs covering cosmology and every search turned up negative; so much for its significance. DL mentioned he was a physicist so perhaps he would be able to address this in more depth.

Ben,
I say that only after reading Feser’s book. In my opinion, it reads like pseudo science and I was asking DL for a second opinion. There is nothing of merit in your comment. However, I don’t expect you to deviate from your MO of launching insults so I’ll just leave it that. I’m not interested in your opinion anyway, as you can notice; I addressed my comment to DL.

ozero91 said...

I didn't know Feser was trying to promote Thomism as science.

"Correct me if I’m wrong, but isn’t Thomism not generally accepted among scientists?"

Is it The Last Superstition that you read? If so, you should know the answer to this, as well as the historical reason.

grodrigues said...

@cautiouslycurious:

"Correct me if I’m wrong, but isn’t Thomism not generally accepted among scientists?"

Is there any reason why scientists have any special competence to judge matters philosophical and metaphysical? And what is this anyway, but if not an appeal to the majority? Is that how you think arguments should be decided?

"In my opinion, it reads like pseudo science and I was asking DL for a second opinion."

Since it does not pretend to pass for science, but metaphysics, how can it be pseudo-science? If you want to see pseudo-science in all its glory just look at Doctor Logic's "arguments".

Anonymous said...

Ozero,

“I didn't know Feser was trying to promote Thomism as science.”
Not particularly. I think the closest he gets is saying that Aquinas views metaphysics, ethics, theology, etc. as sciences, which I take to mean areas of knowledge, which he believes are on par with the sciences that we would consider sciences (physics, biology, etc.). I was talking about the methodology invoked rather than the public relations goal. After all, whether pseudo-science is marketed as science or spiritual, it doesn’t change the dubiousness of the claims.

“Is it The Last Superstition that you read?”
No, his book on Aquinas.

Grodrigues,

“Is there any reason why scientists have any special competence to judge matters philosophical and metaphysical?”

I don’t think I implied that. If you think this is what I’m advocating, then it means that you don’t understand my position.

“And what is this anyway, but if not an appeal to the majority? Is that how you think arguments should be decided?”

It’s not even close to an appeal to the majority. I said demographic X doesn’t take Y seriously because of reason Z. We then went on a tangent about whether or not demographic X takes Y seriously in which no point was being made, just discussing facts about demographic X. If you think that is an appeal to the majority, you don’t know what you’re talking about.

“Since it does not pretend to pass for science, but metaphysics, how can it be pseudo-science?”
I never said it tried to pass as science, I said that it reads **like** pseudo-science, as in, it has a lot of qualities that make pseudo-science pseudo-science.

Son of Ya'Kov said...

cc

You are full of it & you don't have the slightest idea how to argue against AT metaphysics.

ozero91 said...

If you want to know reason Z, read up on the moderns, like Descartes, Kant, Hume, Locke, etc.

Anonymous said...

Ben,

Didn't even try to argue against it. Your insistence to the contrary simply displays your bias.

Anonymous said...

Ben,

Didn't even try to argue against it. Your insistence to the contrary simply displays your bias.

Doctor Logic said...

CC,

I don't think that practicing physicists give a lot of thought to philosophy. However, physicists are pretty good at tracking how much information is in their models, and good at recognizing when one model or formulation is equivalent to another. The relevance to the debate here is that physicists are comfortable with there being some lack of causality (i.e., some events without causes for some of their effects). So, intelligibility is not at stake, as the AT folk like to pretend, unless we're dismissing causality across the board, and no one is doing that. And, I repeat, there's nothing inconsistent about partial determination or the presence of brute facts. Of course, a model with fewer brute facts is preferred, but theists don't have this because they cannot predict any of the brute facts from their premise (as they readily admit). They just substitute theistic brute facts for physical ones for no gain in simplicity (in fact, it makes things worse).

As for the Courtier's Reply, I've noticed that from the AT people, too. I sometimes think they're deliberately keeping their arguments obscure, as if they don't actually want to explain them. It's like they want to arrogantly dismiss us, not explain to us. They don't seem inspired by the arguments, and excited to convey them. Nor do they charitably interpret the arguments of the other side in order to honestly tackle the arguments with which they are presented.

As you have surmised, AT philosophy itself is infantile and obsolete. They ignore Hume on induction, ignore Kant on categories and on inferences to metaphysics, ignore the empiricists and positivists on verifiability, and ignore Wittgenstein on the dangers of taking words out of word games and pretending they apply out of context (e.g., act, potency, cause, existence, etc.). I can see its aesthetic appeal to theists, and people who insist the universe be equally intuitive at human scales, quantum scales, and cosmological scales. However, I can distinguish between philosophies that are pleasing and those that have merit, and they're not always the same.

William said...

" I said demographic X doesn’t take Y seriously because of reason Z."

I see the A-T theorists here blaming modern philosophy for removing the concepts of act/potency, essence/substance, or other A-T intuitions from the default tools of people other than those who have studied their subject.

A-T theorists here may believe that modernist metaphysics has kept people from the proper intuitions. But I think it's more likely that the Aristotelian intuitions were only "natural" when Aristotelian explanation was used for ordinary commonsense explanations of natural phenomena in the outside world.

Remember Aristotle never used the term "metaphysics," but only "physics" in his discussions of act/potency. I doubt he ever saw the metaphysical/physical distinction that keeps our A-T theorists safe from consequences of current science's domination of the explanations for natural phenomena (no need to revise the Five Ways, for example).

Aristotelian natural explanations for natural phenomena have been surpassed for centuries by better things. As a consequence, most non-philosophers have no prior contact with any reason to have act/potency intuitions. They still have to deal with natural phenomena, but they have better tools for guiding their intuitions about the natural phenomena than Aristotle had.

The lack of intuitive grasp of A-T theory is then due to Aristotle's ideas about physics and biology (not first philosophy or theology) based on the act/potency distinction having been almost entirely obsoleted, so non-philosophers simply use more useful concepts in their everyday activities.

That is a sad consequence for A-T theory, coming in from outside its metaphysical boundaries. The loss of those "proper" intuitions is really due to Aristotle's eclipse as a scientist, not the loss of any preeminence of A-T in metaphysics. In that regard, I agree with Dr. Logic and Linton.

Son of Ya'Kov said...
This comment has been removed by the author.
Son of Ya'Kov said...

Additional:

>Remember Aristotle never used the term "metaphysics," but only "physics" in his discussions of act/potency.

^ Tredennick's translation, with links to his footnote cross references, using the Perseus online resources: "For we say that both that which sees potentially and that which sees actually is "a seeing thing." And in the same way we call "understanding" both that which can use the understanding, and that which does ; and we call "tranquil" both that in which tranquillity is already present, and that which is potentially tranquil. Similarly too in the case of substances. For we say that Hermes is in the stone, ([Cf. Aristotle Met. 3.5.6.) and the half of the line in the whole; and we call "corn" what is not yet ripe. But when a thing is potentially existent and when not, must be defined elsewhere." Aristotle Metaphysics 9.9</n.

Feser was right. Modern philosophers know little or nothing about the classics.

Son of Ya'Kov said...

@William

My previous post was more accusatory but after reading you again I though better of it & as you can see took it down.

Never the less I find your statement "Aristotle never used the term "metaphysics," but only "physics" in his discussions of act/potency." to be prima faca false.

Son of Ya'Kov said...

>They ignore Hume on induction, ignore Kant on categories and on inferences to metaphysics, ignore the empiricists and positivists on verifiability, and ignore Wittgenstein on the dangers of taking words out of word games and pretending they apply out of context (e.g., act, potency, cause, existence, etc.).

Hume and Kant where wrong their philosophies are inconsistent. Indeed reading Feser, Oderberg and others they showed they are deeply flawed.

You DL hold them dogmatically and uncritically which is why you are pointless.

Papalinton said...

From Ben: "Feser was right. Modern philosophers know little or nothing about the classics."

This brings a smile to my lips. It also jogs my memory of the story about a proud father attending a marching-out ceremony to watch his son graduate from a military academy. The proud parent gesticulates, pointing out and loudly informing everyone, "Oh! Look! There's my son! The only one in step."

Quite some hero-worship for Feser, has Ben; a Feser Fetish. ;o)


Son of Ya'Kov said...

whatever you say Peggy Hill.

grodrigues said...

@cautiouslycurious:

"I think the closest he gets is saying that Aquinas views metaphysics, ethics, theology, etc. as sciences, which I take to mean areas of knowledge, which he believes are on par with the sciences that we would consider sciences (physics, biology, etc.)."

Feser is using "science" in the same sense Aristotle used the term, not in the modern sense.

" I said demographic X doesn’t take Y seriously because of reason Z."

Demographic X (scientists) is in general ignorant of Y (thomism), much less have an informed opinion on it. But I will repeat my question; suppose X did have an opinion on Y, and assume also it was an unfavorable one. So what? They have no special competence to judge matters philosophical, at least none beyond your average non-specialist.

"I said that it reads **like** pseudo-science, as in, it has a lot of qualities that make pseudo-science pseudo-science."

In other words, surprise surprise, you do not have the faintest idea of what you are talking about. And this also includes science, of which the Gnus here like to abuse to serve their their own needs.

grodrigues said...

@William:

"I see the A-T theorists here blaming modern philosophy for removing the concepts of act/potency, essence/substance, or other A-T intuitions from the default tools of people other than those who have studied their subject."

The problems with the typically modern rejection of the Aristotelian do not lie in people using other tools do their job. Of course, scientists (in the modern sense) will have little use in their daily job for the notions of act and potency; there is nothing bizarre, surprising, or unusual in this. It follows inevitably from a consideration of the divisions in the sciences (now in the Aristotelian sense), from their proper subject matter and methods.

"A-T theorists here may believe that modernist metaphysics has kept people from the proper intuitions. But I think it's more likely that the Aristotelian intuitions were only "natural" when Aristotelian explanation was used for ordinary commonsense explanations of natural phenomena in the outside world."

The problem is not whether "modernist metaphysics" (whatever you mean by this exactly) having "kept people from the proper intuitions". There is a Thomist prejudice in favor of common sense (which I will not bother to justify here), but this prejudice is defeasible on the face of good arguments. The problem with the rejection of the Aristotelian notions lies elsewhere and is much more serious, and much more interesting, than keeping "people from the proper intuitions". It also lies in the *reasons* for the rejection.

"I doubt he ever saw the metaphysical/physical distinction that keeps our A-T theorists safe from consequences of current science's domination of the explanations for natural phenomena (no need to revise the Five Ways, for example)."

Ah the old stand-by that metaphysical explanations are shielded from empirical discoveries...

First, there are no empirical discoveries that have forced a revision of the basic Aristotelian notions, at best only a refinement (I know of no such cases, so the refinement thingy is just a confession of my ignorance). Second, empirical discoveries *cannot* force by themselves a revision of metaphysical notions, because such discoveries are the data to be explained, and their theoretical interpretation is itself suffused with metaphysical assumptions. From this it follows, that the battle is engaged in the metaphysical field. You want to dispute the distinction between act and potency? Then offer a competing metaphysical account of change. What will the empirical sciences will do for you here? Nothing.

"Aristotelian natural explanations for natural phenomena have been surpassed for centuries by better things."

No one disputes this.

"As a consequence, most non-philosophers have no prior contact with any reason to have act/potency intuitions. They still have to deal with natural phenomena, but they have better tools for guiding their intuitions about the natural phenomena than Aristotle had."

Since metaphysical explanations (or more properly speaking, philosophy of nature) do not aim at explaining "natural phenomena" but at explaining the most general features of beings that make the empirical sciences possible in the first place, your objection is, like your whole comment, just an irrelevant disquisition borne out of ignorance. But we have already gone through this and you should know better. Apparently you do not. Oh well.

"That is a sad consequence for A-T theory, coming in from outside its metaphysical boundaries."

Thanks for your heartfelt condolences. In the spirit of "let the dead bury their dead", you should first tend to the grave you are lying in though, seeing the disarray it is in.

Anonymous said...

DL,
Thanks for your reply. I think this touched onto what I was trying to get at:

“I sometimes think they're deliberately keeping their arguments obscure, as if they don't actually want to explain them. It's like they want to arrogantly dismiss us, not explain to us. They don't seem inspired by the arguments, and excited to convey them.”

When I was reading through these terms, it seemed like they were ‘stretchable definitions’ like the term “kind” that can be expanded or narrowed depending on the purposes of the speaker. It’s almost like the creationist saying that things only come from their own kind and then using that to reach their conclusion that evolution can’t happen. In comparison, it appeared that essences and final causes are only extensions of the speaker. For example, when it comes to homosexuality, it’s not obvious that the essence of humans has anything to do with our intentions to reproduce and it’s not obvious that the final cause of homosexuals is anything contrary to forming same-sex unions, yet this is somehow objectively determined? How does one go from “the essence of a thing is signified by its definition” to “gay marriage destroys the very essence of the human creature”? Also, it’s not obvious that not conforming to the essence of what someone else decides is in anyway bad either, and it’s not even argued. There is no methodology explained for determining the final cause or essence of a thing, rather, it is just declared. As such, it looks like trivially obvious statements are being stretched to support a priori beliefs.

“They just substitute theistic brute facts for physical ones for no gain in simplicity (in fact, it makes things worse).”

Yeah, this is annoying too, as if stipulating that a thing is immaterial (i.e. doesn’t have parts) automatically zeros out its Kolmogorov complexity.

Anonymous said...

Grodrigues,
“Feser is using "science" in the same sense Aristotle used the term, not in the modern sense.”
And? I explicitly mentioned he didn’t use it in the modern sense. Check your blind spots.

“Demographic X (scientists) is in general ignorant of Y (thomism), much less have an informed opinion on it. But I will repeat my question; suppose X did have an opinion on Y, and assume also it was an unfavorable one. So what? They have no special competence to judge matters philosophical, at least none beyond your average non-specialist.”

I only mentioned it because it’s my impression that he thinks scientists should be more accepting of Thomism, but he doesn’t understand why they don’t pay attention to it in the first place. He makes the same complaints to modern philosophers as well. Are you going to say that modern philosophers also don’t have said special competence? Anyway, the special competence of scientists is that they are well adept at separating the objective from the subjective, or as DL puts it, the ideas that are pleasing to us and those that have merit.

“In other words, surprise surprise, you do not have the faintest idea of what you are talking about. And this also includes science, of which the Gnus here like to abuse to serve their their own needs.”

Surprise, surprise, you’re dismissing criticism out of hand. If you’re going to put forth the effort to respond, you should at least put in some substance along with it.

ozero91 said...

What, no mention of Cartwright and friends?

«Oldest ‹Older   1 – 200 of 259   Newer› Newest»